Как найти соотношение давлений

Оглавление:

  • Основные теоретические сведения
    • Давление. Закон Паскаля. Гидростатическое давление
    • Сообщающиеся сосуды
    • Гидравлический пресс
    • Закон Архимеда. Вес тела в жидкости
    • Плавание тел

Основные теоретические сведения

Давление. Закон Паскаля. Гидростатическое давление

К оглавлению…

Основным отличием жидкостей от твердых (упругих) тел является способность легко изменять свою форму. Части жидкости могут свободно сдвигаться, перемещаясь друг относительно друга. Поэтому жидкость принимает форму сосуда, в который она налита. В жидкость, как и в газообразную среду, можно погружать твердые тела. В отличие от газов жидкости практически несжимаемы. На тело, погруженное в жидкость или газ, действуют силы, распределенные по поверхности тела. Для описания таких распределенных сил в гидростатике вводится новая физическая величина – давление.

Давление определяется как отношение модуля силы F, действующей перпендикулярно поверхности, к площади S этой поверхности:

Формула Давление

Если же сила направлена под некоторым углом к перпендикуляру к площадке, то создаваемое этой силой давление находится по формуле:

Формула Давление

В системе СИ давление измеряется в паскалях (Па): 1 Па = 1 Н/м2. Часто используются внесистемные единицы: нормальное атмосферное давление (атм) и давление одного миллиметра ртутного столба (мм.рт.ст.):

1 атм = 101325 Па = 760 мм.рт.ст.

Закон Паскаля: давление, оказываемое на жидкость (или, к слову, газ), передается в любую точку этой жидкости без изменений и во всех направлениях.

Давление жидкости на дно или боковые стенки сосуда зависит от высоты столба жидкости над той точкой в которой измеряется давление. Гидростатическое давление столба жидкости рассчитывается по формуле:

Формула Давление столба жидкости

Обратите внимание, что оказываемое давление никоим образом не зависит от формы сосуда, а зависит только от рода жидкости (т.е. её плотности) и от высоты столба этой жидкости. Такое же давление на глубине h в соответствии с законом Паскаля жидкость оказывает и на боковые стенки сосуда.

Итак, если в задаче по гидростатике идет речь о давлении столба жидкости на боковую грань в некоторой конкретной точке, то такое давление находится по предыдущей формуле, где h – расстояние от этой точки до поверхности жидкости. Но иногда в задачах по гидростатике необходимо рассчитать среднее давление на всю боковую поверхность сосуда. В таком случае применим формулу:

Формула Давление на боковую поверхность

В этом случае, h – это общая высота столба жидкости в сосуде.

Если жидкость находится в цилиндре под поршнем, то действуя на поршень некоторой внешней силой F, можно создавать в жидкости дополнительное давление p0 = F/S, где: S – площадь поршня. Таким образом, полное давление в жидкости на глубине h можно записать в виде:

Формула Давление на глубине

Если поршень убрать, то давление на поверхность жидкости будет равно атмосферному давлению. Если мы погружаемся в воду, то давление на некоторой глубине тоже будет состоять из двух давлений – давления атмосферы и давления столба воды (которое определяется глубиной погружения).

Сообщающиеся сосуды

К оглавлению…

Сообщающимися называют сосуды, имеющие между собой канал, заполненный жидкостью. Наблюдения показывают, что в сообщающихся сосудах любой формы однородная жидкость всегда устанавливается на одном уровне. задачи на сообщающиеся сосуды очень распространены в гидростатике.

Иначе ведут себя разнородные жидкости даже в одинаковых по форме и размерам сообщающихся сосудах. Дело в том, что в сообщающихся сосудах должно устанавливаться одинаковое давление на одной и той же высоте во всех частях сосуда. Но если жидкости различные, то и высота столбов этих жидкостей должна быть различной, чтобы создать одинаковое давление. Поэтому, разнородные жидкости в сообщающихся сосудах могут и не устанавливаться на одном уровне.

Алгоритм решения задач по гидростатике на сообщающиеся сосуды:

  1. Сделать рисунок.
  2. Выбрать горизонтальный уровень, ниже которого во всех сосудах находится одинаковая жидкость. Если такого уровня нет, то, естественно, за нулевой уровень выбираем дно сосудов.
  3. Записать давления относительно этого уровня во всех сосудах и приравнять.
  4. При необходимости использовать свойство несжимаемости жидкости (объем жидкости, вытекающей из одного сосуда, равен объему жидкости, втекающей в другой сосуд).
  5. Решить математически полученную систему уравнений.

Гидравлический пресс

К оглавлению…

Если оба вертикально расположенных цилиндра сообщающихся сосудов закрыть поршнями, то с помощью внешних сил, приложенных к поршням, в жидкости можно создать большое давление p, во много раз превышающее гидростатическое давление ρgh в любой точке системы. Тогда можно считать, что во всей системе устанавливается одинаковое давление p (согласно закону Паскаля). Если поршни имеют разные площади S1 и S2, то на них со стороны жидкости действуют разные силы F1 = pS1 и F2 = pS2. Такие же по модулю, но противоположно направленные внешние силы должны быть приложены к поршням для удержания системы в равновесии. Таким образом, для гидравлического пресса имеем формулу:

Формула Соотношение равенство давлений

Это соотношение вытекает из равенства давлений и выполняется только в идеальном гидравлическом прессе, т.е. таком в котором нет трения. Если S2 >> S1, то и F2 >> F1. Устройства в которых выполняются эти условия называют гидравлическими прессами (машинами, домкратами). Они позволяют получить значительный выигрыш в силе. Если поршень в узком цилиндре переместить вниз под действием внешней силы F1 на расстояние h1, то поршень в широком цилиндре переместится на расстояние h2, которое может быть найдено из соотношения:

Формула Соотношение равенство объёмов

Данное соотношение вытекает из равенства объемов и выполняется в любом гидравлическом прессе. Это выражение получается потому, что при перемещении поршня перемещаются одинаковые объемы жидкости, то есть сколько жидкости ушло из одного цилиндра столько же пришло во второй, или V1 = V2. Таким образом, выигрыш в силе обязательно сопровождается таким же проигрышем в расстоянии. При этом произведение силы на расстояние остается неизменным:

Формула Соотношение равенство работ

Последняя формула вытекает из равенства работ и выполняется только для идеальных машин, в которых не действуют силы трения. Таким образом, в гидравлическом прессе всё происходит в полном соответствии с «золотым правилом механики»: во сколько раз мы выигрываем в силе, во столько же раз мы проигрываем в расстоянии. При этом ни одна машина не может дать выигрыша в работе.

Так как гидравлический пресс является механизмом, то его работу можно характеризовать КПД (коэффициентом полезного действия). КПД гидравлического пресса в задачах по гидростатике рассчитывается по следующей формуле:

Формула КПД для неидеального гидравлического пресса

где: Апол = F2h2 – полезная работа (работа по подъему груза), Азатр = F1h1 – затраченная работа. В большинстве задач КПД гидравлического пресса принимают за 100%. КПД рассчитывается в том случае, если речь идет о неидеальном гидравлическом прессе.

Еще раз подчеркнем, что для неидеального гидравлического пресса выполняется только соотношение, вытекающее из равенства объемов вытесненной жидкости, а также для таких прессов рассчитывается КПД. Остальные соотношения из этого раздела выполняются только для идеального гидравлического пресса.

Закон Архимеда. Вес тела в жидкости

К оглавлению…

Из–за разности давлений в жидкости на разных уровнях возникает выталкивающая или Архимедова сила, которая вычисляется по формуле:

Формула Сила Архимеда

где: V – объем вытесненной телом жидкости, или же объем погружённой в жидкость части тела, ρ – плотность жидкости в которую погружено тело, и следовательно, ρV – масса вытесненной жидкости.

Архимедова сила, действующая на погруженное в жидкость (или газ) тело, равна весу жидкости (или газа), вытесненной телом. Это утверждение, называемое законом Архимеда, справедливо для тел любой формы.

При этом вес тела (т.е. сила с которой тело действует на опору или подвес) погруженного в жидкость уменьшается. Если принять, что вес покоящегося тела в воздухе равен mg, а именно так мы и будем поступать в большинстве задач (хотя вообще говоря на тело в воздухе также действует очень маленькая сила Архимеда со стороны атмосферы, ведь тело погружено в газ из атмосферы), то для веса тела в жидкости можно легко вывести следующую важную формулу:

Формула Вес тела в жидкости

Эта формула может быть использована при решении большого количества задач. Ее можно запомнить. При помощи закона Архимеда осуществляется не только мореплавание, но и воздухоплавание. Из закона Архимеда вытекает, что если средняя плотность тела ρт больше плотности жидкости (или газа) ρ (или по–другому mg > FA), тело будет опускаться на дно. Если же ρт < ρ (или по–другому mg < FA), тело будет плавать на поверхности жидкости. Объем погруженной части тела будет таков, что вес вытесненной жидкости равен весу тела. Для подъема воздушного шара в воздухе его вес должен быть меньше веса вытесненного воздуха. Поэтому воздушные шары заполняют легкими газами (водородом, гелием) или нагретым воздухом.

Плавание тел

К оглавлению…

Если тело находится на поверхности жидкости (плавает), то на него действует всего две силы (Архимеда вверх и тяжести вниз), которые уравновешивают друг друга. Если тело погружено только в одну жидкость, то записав второй закон Ньютона для такого случая и выполнив простые математические операции можем получить следующее выражение связывающее объемы и плотности:

Условие плавания тела

где: Vпогр – объем погруженной части тела, V – полный объем тела. При помощи этого соотношения легко решается большинство задач на плавание тел.

Давление. Единицы давления.

Человек на лыжах, и без них.

По рыхлому снегу человек идёт с большим трудом, глубоко проваливаясь при каждом шаге. Но, надев лыжи, он может идти, почти не проваливаясь в него. Почему? На лыжах или без лыж человек действует на снег с одной и той же силой, равной своему весу. Однако действие этой силы в обоих случаях различно, потому что различна площадь поверхности, на которую давит человек, с лыжами и без лыж. Площадь поверхности лыж почти в 20 раз больше площади подошвы. Поэтому, стоя на лыжах, человек действует на каждый квадратный сантиметр площади поверхности снега с силой, в 20 раз меньшей, чем стоя на снегу без лыж.

Ученик, прикалывая кнопками газету к доске, действует на каждую кнопку с одинаковой силой. Однако кнопка, имеющая более острый конец, легче входит в дерево.

Значит, результат действия силы зависит не только от её модуля, направления и точки приложения, но и от площади той поверхности, к которой она приложена (перпендикулярно которой она действует).

Этот вывод подтверждают физические опыты.

Опыт.Результат действия данной силы зависит от того, какая сила действует на единицу площади поверхности.

По углам небольшой доски надо вбить гвозди. Сначала гвозди, вбитые в доску, установим на песке остриями вверх и положим на доску гирю. В этом случае шляпки гвоздей лишь незначительно вдавливаются в песок. Затем доску перевернем и поставим гвозди на острие. В этом случае площадь опоры меньше, и под действием той же силы гвозди значительно углубляются в песок.

Опыт. Вторая иллюстрация.

От того, какая сила действует на каждую единицу площади поверхности, зависит результат действия этой силы.

В рассмотренных примерах силы действовали перпендикулярно поверхности тела. Вес человека был перпендикулярен поверхности снега; сила, действовавшая на кнопку, перпендикулярна поверхности доски.

Величина, равная отношению силы, действующей перпендикулярно поверхности, к площади этой поверхности, называется давлением.

Чтобы определить давление, надо силу, действующую перпендикулярно поверхности, разделить на площадь поверхности:

давление = сила / площадь.

Обозначим величины, входящие в это выражение: давление — p, сила, действующая на поверхность, — F и площадь поверхности — S.

Тогда получим формулу:

p = F/S

Понятно, что бóльшая по значению сила, действующую на ту же площадь, будет производить большее давление.

За единицу давления принимается такое давление, которое производит сила в 1 Н, действующая на поверхность площадью 1 м2 перпендикулярно этой поверхности.

Единица давления — ньютон на квадратный метр ( 1 Н / м2 ). В честь французского ученого Блеза Паскаля она называется паскалем (Па). Таким образом,

1 Па = 1 Н / м2 .

Используется также другие единицы давления: гектопаскаль (гПа) и килопаскаль (кПа).

1 кПа = 1000 Па;

1 гПа = 100 Па;

1 Па = 0,001 кПа;

1 Па = 0,01 гПа.

Пример. Рассчитать давление, производимое на пол мальчиком, масса которого 45 кг, а площадь подошв его ботинок, соприкасающихся с полом, равна 300 см2.

Запишем условие задачи и решим её.

Дано: m = 45 кг, S = 300 см2; p = ?

В единицах СИ: S = 0,03 м2

Решение:

p = F/S,

F = P,

P = g·m,

P = 9,8 Н · 45 кг ≈ 450 Н,

p = 450/0,03 Н / м2 = 15000 Па = 15 кПа

‘Ответ’: p = 15000 Па = 15 кПа

Способы уменьшения и увеличения давления.

Тяжелый гусеничный трактор производит на почву давление равное 40 — 50 кПа, т. е. всего в 2 — 3 раза больше, чем давление мальчика массой 45 кг. Это объясняется тем, что вес трактора распределяется на бóльшую площадь за счёт гусеничной передачи. А мы установили, что чем больше площадь опоры, тем меньше давление, производимое одной и той же силой на эту опору.

В зависимости от того, нужно ли получить малое или большое давление, площадь опоры увеличивается или уменьшается. Например, для того, чтобы грунт мог выдержать давление возводимого здания, увеличивают площадь нижней части фундамента.

Шины грузовых автомобилей и шасси самолетов делают значительно шире, чем легковых. Особенно широкими делают шины у автомобилей, предназначенных для передвижения в пустынях.

Тяжелые машины, как трактор, танк или болотоход, имея большую опорную площадь гусениц, проходят по болотистой местности, по которой не пройдет человек.

С другой стороны, при малой площади поверхности можно небольшой силой произвести большое давление. Например, вдавливая кнопку в доску, мы действуем на нее с силой около 50 Н. Так как площадь острия кнопки примерно 1 мм2, то давление, производимое ею, равно:

p = 50 Н/ 0, 000 001 м2 = 50 000 000 Па = 50 000 кПа.

Для сравнения, это давление в 1000 раз больше давления, производимого гусеничным трактором на почву. Можно найти еще много таких примеров.

Лезвие режущих и острие колющих инструментов (ножей, ножниц, резцов, пил, игл и др.) специально остро оттачивается. Заточенный край острого лезвия имеет маленькую площадь, поэтому при помощи даже малой силы создается большое давление, и таким инструментом легко работать.

Режущие и колющие приспособления встречаются и в живой природе: это зубы, когти, клювы, шипы и др. — все они из твердого материала, гладкие и очень острые.

Давление

Известно, что молекулы газа беспорядочно движутся.
Опыт. Здесь мы узнаем, что газ давит на стенки сосуда по всем направлениям одинаково.

Мы уже знаем, что газы, в отличие от твердых тел и жидкостей, заполняют весь сосуд, в котором находятся. Например, стальной баллон для хранения газов, камера автомобильной шины или волейбольный мяч. При этом газ оказывает давление на стенки, дно и крышку баллона, камеры или любого другого тела, в котором он находится. Давление газа обусловлено иными причинами, чем давление твердого тела на опору.

Известно, что молекулы газа беспорядочно движутся. При своем движении они сталкиваются друг с другом, а также со стенками сосуда, в котором находится газ. Молекул в газе много, поэтому и число их ударов очень велико. Например, число ударов молекул воздуха, находящегося в комнате, о поверхность площадью 1 см2 за 1 с выражается двадцатитрехзначным числом. Хотя сила удара отдельной молекулы мала, но действие всех молекул на стенки сосуда значительно, — оно и создает давление газа.

Итак, давление газа на стенки сосуда (и на помещенное в газ тело) вызывается ударами молекул газа.

Рассмотрим следующий опыт. Под колокол воздушного насоса поместим резиновый шарик. Он содержит небольшое количество воздуха и имеет неправильную форму. Затем насосом откачиваем воздух из-под колокола. Оболочка шарика, вокруг которой воздух становится все более разреженным, постепенно раздувается и принимает форму правильного шара.

Как объяснить этот опыт?

Для хранения и перевозки сжатого газа используются специальные прочные стальные баллоны.

В нашем опыте движущиеся молекулы газа непрерывно ударяют о стенки шарика внутри и снаружи. При откачивании воздуха число молекул в колоколе вокруг оболочки шарика уменьшается. Но внутри шарика их число не изменяется. Поэтому число ударов молекул о внешние стенки оболочки становится меньше, чем число ударов о внутренние стенки. Шарик раздувается до тех пор, пока сила упругости его резиновой оболочки не станет равной силе давления газа. Оболочка шарика принимает форму шара. Это показывает, что газ давит на ее стенки по всем направлениям одинаково. Иначе говоря, число ударов молекул, приходящихся на каждый квадратный сантиметр площади поверхности, по всем направлениям одинаково. Одинаковое давление по всем направлениям характерно для газа и является следствием беспорядочного движения огромного числа молекул.

Попытаемся уменьшить объем газа, но так, чтобы масса его осталась неизменной. Это значит, что в каждом кубическом сантиметре газа молекул станет больше, плотность газа увеличится. Тогда число ударов молекул о стенки увеличится, т. е. возрастет давление газа. Это можно подтвердить опытом.

На рисунке а изображена стеклянная трубка, один конец которой закрыт тонкой резиновой пленкой. В трубку вставлен поршень. При вдвигании поршня объем воздуха в трубке уменьшается, т. е. газ сжимается. Резиновая пленка при этом выгибается наружу, указывая на то, что давление воздуха в трубке увеличилось.

Наоборот, при увеличении объема этой же массы газа, число молекул в каждом кубическом сантиметре уменьшается. От этого уменьшится число ударов о стенки сосуда — давление газа станет меньше. Действительно, при вытягивании поршня из трубки объем воздуха увеличивается, пленка прогибается внутрь сосуда. Это указывает на уменьшение давления воздуха в трубке. Такие же явления наблюдались бы, если бы вместо воздуха в трубке находился бы любой другой газ.

Итак, при уменьшении объема газа его давление увеличивается, а при увеличении объема давление уменьшается при условии, что масса и температура газа остаются неизменными.

А как изменится давление газа, если нагреть его при постоянном объеме? Известно, что скорость движения молекул газа при нагревании увеличивается. Двигаясь быстрее, молекулы будут ударять о стенки сосуда чаще. Кроме того, каждый удар молекулы о стенку будет сильнее. Вследствие этого, стенки сосуда будут испытывать большее давление.

Следовательно, давление газа в закрытом сосуде тем больше, чем выше температура газа, при условии, что масса газа и объем не изменяются.

Из этих опытов можно сделать общий вывод, что давление газа тем больше, чем чаще и сильнее молекулы ударяют о стенки сосуда.

Для хранения и перевозки газов их сильно сжимают. При этом давление их возрастает, газы необходимо заключать в специальные, очень прочные баллоны. В таких баллонах, например, содержат сжатый воздух в подводных лодках, кислород, используемый при сварке металлов. Конечно же, мы должны навсегда запомнить, что газовые баллоны нельзя нагревать, тем более, когда они заполнены газом. Потому что, как мы уже понимаем, может произойти взрыв с очень неприятными последствиями.

Закон Паскаля.

Давление передается в каждую точку жидкости или газа.
Давление поршня передается в каждую точку жидкости, заполняющей шар.
Теперь газ.

В отличие от твердых тел отдельные слои и мелкие частицы жидкости и газа могут свободно перемещаться относительно друг друга по всем направлениям. Достаточно, например, слегка подуть на поверхность воды в стакане, чтобы вызвать движение воды. На реке или озере при малейшем ветерке появляется рябь.

Подвижностью частиц газа и жидкости объясняется, что давление, производимое на них, передается не только в направлении действия силы, а в каждую точку. Рассмотрим это явление подробнее.

На рисунке, а изображен сосуд, в котором содержится газ (или жидкость). Частицы равномерно распределены по всему сосуду. Сосуд закрыт поршнем, который может перемещаться вверх и вниз.

Прилагая некоторую силу, заставим поршень немного переместиться внутрь и сжать газ (жидкость), находящийся непосредственно под ним. Тогда частицы (молекулы) расположатся в этом месте более плотно, чем прежде(рис, б). Благодаря подвижности частицы газа будут перемещаться по всем направлениям. Вследствие этого их расположение опять станет равномерным, но более плотным, чем раньше (рис, в). Поэтому давление газа всюду возрастет. Значит, добавочное давление передается всем частицам газа или жидкости. Так, если давление на газ (жидкость) около самого поршня увеличится на 1 Па, то во всех точках внутри газа или жидкости давление станет больше прежнего на столько же. На 1 Па увеличится давление и на стенки сосуда, и на дно, и на поршень.

Давление, производимое на жидкость или газ, передается на любую точку одинаково во всех направлениях.

Это утверждение называется законом Паскаля.

На основе закона Паскаля легко объяснить следующие опыты.

На рисунке изображен полый шар, имеющий в различных местах небольшие отверстия. К шару присоединена трубка, в которую вставлен поршень. Если набрать воды в шар и вдвинуть в трубку поршень, то вода польется из всех отверстий шара. В этом опыте поршень давит на поверхность воды в трубке. Частицы воды, находящиеся под поршнем, уплотняясь, передают его давление другим слоям, лежащим глубже. Таким образом, давление поршня передается в каждую точку жидкости, заполняющей шар. В результате часть воды выталкивается из шара в виде одинаковых струек, вытекающих из всех отверстий.

Если шар заполнить дымом, то при вдвигании поршня в трубку из всех отверстий шара начнут выходить одинаковые струйки дыма. Это подтверждает, что и газы передают производимое на них давление во все стороны одинаково.

Давление в жидкости и газе.

Под действием веса жидкости резиновое дно в трубке прогнется.

На жидкости, как и на все тела на Земле, действует сила тяжести. Поэтому, каждый слой жидкости, налитой в сосуд, своим весом создает давление, которое по закону Паскаля передается по всем направлениям. Следовательно, внутри жидкости существует давление. В этом можно убедиться на опыте.

В стеклянную трубку, нижнее отверстие которой закрыто тонкой резиновой пленкой, нальем воду. Под действием веса жидкости дно трубки прогнется.

Опыт показывает, что, чем выше столб воды над резиновой пленкой, тем больше она прогибается. Но всякий раз после того, как резиновое дно прогнулось, вода в трубке приходит в равновесие (останавливается), так как, кроме силы тяжести, на воду действует сила упругости растянутой резиновой пленки.

Иллюстрация.
Дно отходит от цилиндра вследствие давления на него силы тяжести.

Опустим трубку с резиновым дном, в которую налита вода, в другой, более широкий сосуд с водой. Мы увидим, что по мере опускания трубки резиновая пленка постепенно выпрямляется. Полное выпрямление пленки показывает, что силы, действующие на нее сверху и снизу, равны. Наступает полное выпрямление пленки тогда, когда уровни воды в трубке и сосуде совпадают.

Такой же опыт можно провести с трубкой, в которой резиновая пленка закрывает боковое отверстие, как это показано на рисунке, а. Погрузим эту трубку с водой в другой сосуд с водой, как это изображено на рисунке, б. Мы заметим, что пленка снова выпрямится, как только уровни воды в трубке и сосуде сравняются. Это означает, что силы, действующие на резиновую пленку, одинаковы со всех сторон.

Возьмем сосуд, дно которого может отпадать. Опустим его в банку с водой. Дно при этом окажется плотно прижатым к краю сосуда и не отпадет. Его прижимает сила давления воды, направленная снизу вверх.

Будем осторожно наливать воду в сосуд и следить за его дном. Как только уровень воды в сосуде совпадет с уровнем воды в банке, оно отпадет от сосуда.

В момент отрыва на дно давит сверху вниз столб жидкости в сосуде, а снизу вверх на дно передается давление такого же по высоте столба жидкости, но находящейся в банке. Оба эти давления одинаковы, дно же отходит от цилиндра вследствие действия на него собственной силы тяжести.

Выше были описаны опыты с водой, но если взять вместо воды любую другую жидкость, результаты опыта будут те же.

Итак, опыты показывают, что внутри жидкости существует давление, и на одном и том же уровне оно одинаково по всем направлениям. С глубиной давление увеличивается.

Газы в этом отношении не отличаются от жидкостей, ведь они тоже имеют вес. Но надо помнить, что плотность газа в сотни раз меньше плотности жидкости. Вес газа, находящегося в сосуде, мал, и его «весовое» давление во многих случаях можно не учитывать.

Расчет давления жидкости на дно и стенки сосуда.

Расчет давления жидкости на дно и стенки сосуда.

Рассмотрим, как можно рассчитывать давление жидкости на дно и стенки сосуда. Решим сначала задачу для сосуда, имеющего форму прямоугольного параллелепипеда.

Сила F, с которой жидкость, налитая в этот сосуд, давит на его дно, равна весу P жидкости, находящейся в сосуде. Вес жидкости можно определить, зная ее массу m. Массу, как известно, можно вычислить по формуле: m = ρ·V. Объем жидкости, налитой в выбранный нами сосуд, легко рассчитать. Если высоту столба жидкости, находящейся в сосуде, обозначить буквой h, а площадь дна сосуда S, то V = S·h.

Масса жидкости m = ρ·V, или m = ρ·S·h .

Вес этой жидкости P = g·m, или P = g·ρ·S·h.

Так как вес столба жидкости равен силе, с которой жидкость давит на дно сосуда, то, разделив вес P на площадь S, получим давление жидкости p:

p = P/S , или p = g·ρ·S·h/S,

то есть

p = g·ρ·h.

Мы получили формулу для расчета давления жидкости на дно сосуда. Из этой формулы видно, что давление жидкости на дно сосуда зависит только от плотности и высоты столба жидкости.

Следовательно, по выведенной формуле можно рассчитывать давление жидкости, налитой в сосуд любой формы (строго говоря, наш расчет годится только для сосудов, имеющих форму прямой призмы и цилиндра. В курсах физики для института доказано, что формула верна и для сосуда произвольной формы). Кроме того, по ней можно вычислить и давление на стенки сосуда. Давление внутри жидкости, в том числе давление снизу вверх, также рассчитывается по этой формуле, так как давление на одной и той же глубине одинаково по всем направлениям.

При расчете давления по формуле p = gρh надо плотность ρ выражать в килограммах на кубический метр (кг/м3), а высоту столба жидкости h — в метрах (м), g = 9,8 Н/кг, тогда давление будет выражено в паскалях (Па).

Пример. Определите давление нефти на дно цистерны, если высота столба нефти 10 м, а плотность ее 800 кг/м3 .

Запишем условие задачи и запишем ее.

Дано:

h = 10 м

ρ = 800 кг/м3

P = ?

Решение:

p = gρh,

p = 9.8 Н/кг · 800 кг/м3 · 10 м ≈ 80 000 Па ≈ 80 кПа.

Ответ: p ≈ 80 кПа.

Сообщающиеся сосуды.

Сообщающиеся сосуды.

На рисунке изображены два сосуда, соединённые между собой резиновой трубкой. Такие сосуды называются сообщающимися. Лейка, чайник, кофейник — примеры сообщающихся сосудов. Из опыта мы знаем, что вода, налитая, например, в лейку, стоит всегда на одном уровне в носике и внутри.

С сообщающимися сосудами можно проделать следующий простой опыт. В начале опыта резиновую трубку зажимаем в середине, и в одну из трубок наливаем воду. Затем зажим открываем, и вода вмиг перетекает в другую трубку, пока поверхности воды в обеих трубках не установятся на одном уровне. Можно закрепить одну из трубок в штативе, а другую поднимать, опускать или наклонять в разные стороны. И в этом случае, как только жидкость успокоится, ее уровни в обеих трубках уравняются.

В сообщающихся сосудах любой формы и сечения поверхности однородной жидкости устанавливаются на одном уровне (при условии, что давление воздуха над жидкостью одинаково) (рис. 109).

Это можно обосновать следующим образом. Жидкость покоится, не перемещаясь из одного сосуда в другой. Значит, давления в обоих сосудах на любом уровне одинаковы. Жидкость в обоих сосудах одна и та же, т. е. имеет одинаковую плотность. Следовательно, должны быть одинаковы и ее высоты. Когда мы поднимаем один сосуд или доливаем в него жидкость, давление в нем увеличивается и жидкость перемещается в другой сосуд до тех пор, пока давления не уравновесятся.

Если в один из сообщающихся сосудов налить жидкость одной плотности, а во второй — другой плотности, то при равновесии уровни этих жидкостей не будут одинаковыми. И это понятно. Мы ведь знаем, что давление жидкости на дно сосуда прямо пропорционально высоте столба и плотности жидкости. А в этом случае плотности жидкостей будут различны.

При равенстве давлений высота столба жидкости с большей плотностью будет меньше высоты столба жидкости с меньшей плотностью (рис.).

Опыт. Как определить массу воздуха.

Вес воздуха. Атмосферное давление.

Существование атмосферного давления.
Атмосферное давление больше, чем давление разреженного воздуха в сосуде.

На воздух, как и на всякое тело, находящееся на Земле, действует сила тяжести, и, значит, воздух обладает весом. Вес воздуха легко вычислить, зная его массу.

На опыте покажем, как вычислить массу воздуха. Для этого нужно взять прочный стеклянный шар с пробкой и резиновой трубкой с зажимом. Выкачаем из него насосом воздух, зажмем трубку зажимом и уравновесим на весах. Затем, открыв зажим на резиновой трубке, впустим в него воздух. Равновесие весов при этом нарушится. Для его восстановления на другую чашку весов придется положить гири, масса которых будет равна массе воздуха в объеме шара.

Опытами установлено, что при температуре 0 °С и нормальном атмосферном давлении масса воздуха объемом 1 м3 равна 1,29 кг. Вес этого воздуха легко вычислить:

P = g·m, P = 9,8 Н/кг · 1,29 кг ≈ 13 Н.

Воздушная оболочка, окружающая Землю, называется атмосфера (от греч. атмос — пар, воздух, и сфера — шар).

Атмосфера, как показали наблюдения за полетом искусственных спутников Земли, простирается на высоту нескольких тысяч километров.

Вследствие действия силы тяжести верхние слои атмосферы, подобно воде океана, сжимают нижние слои. Воздушный слой, прилегающий непосредственно к Земле, сжат больше всего и, согласно закону Паскаля, передает производимое на него давление по всем направлениям.

В результате этого земная поверхность и телá, находящиеся на ней, испытывают давление всей толщи воздуха, или, как обычно говорится в таких случаях, испытывают атмосферное давление.

Существованием атмосферного давления могут быть объяснены многие явления, с которыми мы встречаемся в жизни. Рассмотрим некоторые из них.

На рисунке изображена стеклянная трубка, внутри которой находится поршень, плотно прилегающий к стенкам трубки. Конец трубки опущен воду. Если поднимать поршень, то за ним будет подниматься и вода.

Это явление используется в водяных насосах и некоторых других устройствах.

На рисунке показан цилиндрический сосуд. Он закрыт пробкой, в которую вставлена трубка с краном. Из сосуда насосом откачивается воздух. Затем конец трубки помещается в воду. Если теперь открыть кран, то вода фонтаном брызнет в внутрь сосуда. Вода поступает в сосуд потому, что атмосферное давление больше давления разреженного воздуха в сосуде.

Почему существует воздушная оболочка Земли.

Как и все тела, молекулы газов, входящих в состав воздушной оболочки Земли, притягиваются к Земле.

Но почему же тогда все они не упадут на поверхность Земли? Каким образом сохраняется воздушная оболочка Земли, ее атмосфера? Чтобы понять это, надо учесть, что молекулы газов находятся в непрерывном и беспорядочном движении. Но тогда возникает другой вопрос: почему эти молекулы не улетают в мировое пространство, то есть в космос.

Для того, чтобы совсем покинуть Землю, молекула, как и космический корабль или ракета, должна иметь очень большую скорость (не меньше 11,2 км/с). Это так называемая вторая космическая скорость. Скорость большинства молекул воздушной оболочки Земли значительно меньше этой космической скорости. Поэтому большинство их привязано к Земле силой тяжести, лишь ничтожно малое количество молекул улетает за пределы Земли в космос.

Беспорядочное движение молекул и действие на них силы тяжести приводят в результате к тому, что молекулы газов «парят» в пространстве около Земли, образуя воздушную оболочку, или известную нам атмосферу.

Измерения показывают, что плотность воздуха быстро уменьшается с высотой. Так, на высоте 5,5 км над Землей плотность воздуха в 2 раза меньше его плотность у поверхности Земли, на высоте 11 км — в 4 раза меньше, и т. д. Чем выше, тем воздух разреженнее. И наконец, в самых верхних слоях (сотни и тысячи километров над Землей) атмосфера постепенно переходит в безвоздушное пространство. Четкой границы воздушная оболочка Земли не имеет.

Строго говоря, вследствие действия силы тяжести плотность газа в любом закрытом сосуде неодинакова по всему объему сосуда. Внизу сосуда плотность газа больше, чем в верхних его частях, поэтому и давление в сосуде неодинаково. На дне сосуда оно больше, чем вверху.
Однако для газа, содержащегося в сосуде, это различие в плотности и давлении столь мало, что его можно во многих случаях совсем не учитывать, просто знать об этом. Но для атмосферы, простирающейся на несколько тысяч километров, различие это существенно.

Измерение атмосферного давления. Опыт Торричелли.

Рассчитать атмосферное давление по формуле для вычисления давления столба жидкости (§ 38) нельзя. Для такого расчета надо знать высоту атмосферы и плотность воздуха. Но определенной границы у атмосферы нет, а плотность воздуха на разной высоте различна. Однако измерить атмосферное давление можно с помощью опыта, предложенного в 17 веке итальянским ученым Эванджелиста Торричелли, учеником Галилея.

Опыт Торричелли состоит в следующем: стеклянную трубку длиной около 1 м, запаянную с одного конца, наполняют ртутью. Затем, плотно закрыв второй конец трубки, ее переворачивают и опускают в чашку с ртутью, где под уровнем ртути открывают этот конец трубки. Как и в любом опыте с жидкостью, часть ртути при этом выливается в чашку, а часть ее остается в трубке. Высота столба ртути, оставшейся в трубке, равна примерно 760 мм. Над ртутью внутри трубки воздуха нет, там безвоздушное пространство, поэтому никакой газ не оказывает давления сверху на столб ртути внутри этой трубки и не влияет на измерения.

Опыт Торричелли.

Торричелли, предложивший описанный выше опыт, дал и его объяснение. Атмосфера давит на поверхность ртути в чашке. Ртуть находится в равновесии. Значит, давление в трубке на уровне аа1 (см. рис) равно атмосферному давлению. При изменении атмосферного давления меняется и высота столба ртути в трубке. При увеличении давления столбик удлиняется. При уменьшении давления — столб ртути уменьшает свою высоту.

Давление в трубке на уровне аа1 создается весом столба ртути в трубке, так как в верхней части трубки над ртутью воздуха нет. Отсюда следует, что атмосферное давление равно давлению столба ртути в трубке, т. е.

pатм = pртути .

Измерив высоту столба ртути, можно рассчитать давление, которое производит ртуть. Оно и будет равно атмосферному давлению. Если атмосферное давление уменьшится, то столб ртути в трубке Торричелли понизится.

Чем больше атмосферное давление, тем выше столб ртути в опыте Торричелли. Поэтому на практике атмосферное давление можно измерить высотой ртутного столба (в миллиметрах или сантиметрах). Если, например, атмосферное давление равно 780 мм рт. ст. (говорят «миллиметров ртутного столба»), то это значит, что воздух производит такое же давление, какое производит вертикальный столб ртути высотой 780 мм.

Следовательно, в этом случае за единицу измерения атмосферного давления принимается 1 миллиметр ртутного столба (1 мм рт. ст.). Найдем соотношение между этой единицей и известной нам единицей — паскалем (Па).

Давление столба ртути ρртути высотой 1 мм равно:

p = g·ρ·h, p = 9,8 Н/кг · 13 600 кг/ м3 · 0,001 м ≈ 133,3 Па.

Итак, 1 мм рт. ст. = 133,3 Па.

В настоящее время атмосферное давление принято измерять в гектопаскалях ( 1 гПа = 100 Па). Например, в сводках погоды может быть объявлено, что давление равно 1013 гПа, это то же самое, что 760 мм рт. ст.

Наблюдая ежедневно за высотой ртутного столба в трубке, Торричелли обнаружил, что эта высота меняется, т. е. атмосферное давление непостоянно, оно может увеличиваться и уменьшаться. Торричелли заметил также, что атмосферное давление связано с изменением погоды.

Если к трубке с ртутью, использовавшейся в опыте Торричелли, прикрепить вертикальную шкалу, то получится простейший прибор — ртутный барометр (от греч. барос — тяжесть, метрео — измеряю). Он служит для измерения атмосферного давления.

Барометр — анероид.

В практике для измерения атмосферного давления используют металлический барометр, называемый анероидом (в переводе с греческого — безжидкостный). Так барометр называют потому, что в нем нет ртути.

Внешний вид анероида изображен на рисунке. Главная часть его — металлическая коробочка 1 с волнистой (гофрированной) поверхностью (см. др. рис.). Из этой коробочки выкачан воздух, а чтобы атмосферное давление не раздавило коробочку, ее крышка 2 пружиной оттягивается вверх. При увеличении атмосферного давления крышка прогибается вниз и натягивает пружину. При уменьшении давления пружина выпрямляет крышку. К пружине с помощью передаточного механизма 3 прикреплена стрелка-указатель 4, которая продвигается вправо или влево при изменении давления. Под стрелкой укреплена шкала, деления которой нанесены по показаниям ртутного барометра. Так, число 750, против которого стоит стрелка анероида (см. рис.), показывает, что в данный момент в ртутном барометре высота ртутного столба 750 мм.

Следовательно, атмосферное давление равно 750 мм рт. ст. или ≈ 1000 гПа.

Значение атмосферного давления весьма важно для предвидения погоды на ближайшие дни, так как изменение атмосферного давления связано с изменением погоды. Барометр — необходимый прибор для метеорологических наблюдений.

Атмосферное давление на различных высотах.

В жидкости давление, как мы знаем, зависит от плотности жидкости и высоты ее столба. Вследствие малой сжимаемости плотность жидкости на различных глубинах почти одинакова. Поэтому, вычисляя давление, мы считаем ее плотность постоянной и учитываем только изменение высоты.

Сложнее дело обстоит с газами. Газы сильно сжимаемы. А чем сильнее газ сжат, тем больше его плотность, и тем большее давление он производит. Ведь давление газа создается ударами его молекул о поверхность тела.

Слои воздуха у поверхности Земли сжаты всеми вышележащими слоями воздуха, находящимися над ними. Но чем выше от поверхности слой воздуха, тем слабее он сжат, тем меньше его плотность. Следовательно, тем меньшее давление он производит. Если, например, воздушный шар поднимается над поверхностью Земли, то давление воздуха на шар становиться меньше. Это происходит не только потому, что высота столба воздуха над ним уменьшается, но еще и потому, что уменьшается плотность воздуха. Вверху она меньше, чем внизу. Поэтому зависимость давления воздуха от высоты сложнее, чем жидкости.

Наблюдения показывают, что атмосферное давление в местностях, лежащих на уровне моря, в среднем равно 760 мм рт. ст.

Атмосферное давление, равное давлению столба ртути высотой 760 мм при температуре 0 °С, называется нормальным атмосферным давлением.

Нормальное атмосферное давление равно 101 300 Па = 1013 гПа.

Чем больше высота над уровнем моря, тем давление меньше.

При небольших подъемах, в среднем, на каждые 12 м подъема давление уменьшается на 1 мм рт. ст. (или на 1,33 гПа).

Зная зависимость давления от высоты, можно по изменению показаний барометра определить высоту над уровнем моря. Анероиды, имеющие шкалу, по которой непосредственно можно измерить высоту над уровнем моря, называются высотомерами. Их применяют в авиации и при подъеме на горы.

Манометры.

Мы уже знаем, что для измерения атмосферного давления применяют барометры. Для измерения давлений, бóльших или меньших атмосферного, используется манометры (от греч. манос — редкий, неплотный, метрео — измеряю). Манометры бывают жидкостные и металлические.

Рассмотрим сначала устройство и действие открытого жидкостного манометра. Он состоит из двухколенной стеклянной трубки, в которую наливается какая-нибудь жидкость. Жидкость устанавливается в обоих коленах на одном уровне, так как на ее поверхность в коленах сосуда действует только атмосферное давление.

Чтобы понять, как работает такой манометр, его можно соединить резиновой трубкой с круглой плоской коробкой, одна сторона которой затянута резиновой пленкой. Если надавить пальцем на пленку, то уровень жидкости в колене манометра, соединенном в коробкой, понизится, а в другом колене повысится. Чем это объясняется?

При надавливании на пленку увеличивается давление воздуха в коробке. По закону Паскаля это увеличение давления передается и жидкости в том колене манометра, которое присоединено к коробке. Поэтому давление на жидкость в этом колене будет больше, чем в другом, где на жидкость действует только атмосферное давление. Под действием силы этого избыточного давления жидкость начнет перемещаться. В колене со сжатым воздухом жидкость опустится, в другом — поднимется. Жидкость придет в равновесие (остановится), когда избыточное давление сжатого воздуха уравновесится давлением, которое производит избыточный столб жидкости в другом колене манометра.

Чем сильнее давить на пленку, тем выше избыточный столб жидкости, тем больше его давление. Следовательно, об изменении давления можно судить по высоте этого избыточного столба.

На рисунке показано, как таким манометром можно измерять давление внутри жидкости. Чем глубже погружается в жидкость трубочка, тем больше становится разность высот столбов жидкости в коленах манометра, тем, следовательно, и большее давление производит жидкость.

Если установить коробочку прибора на какой-нибудь глубине внутри жидкости и поворачивать ее пленкой вверх, вбок и вниз, то показания манометра при этом не будут меняется. Так и должно быть, ведь на одном и том же уровне внутри жидкости давление одинаково по всем направлениям.

На рисунке изображен металлический манометр. Основная часть такого манометра — согнутая в трубу металлическая трубка 1, один конец которой закрыт. Другой конец трубки с помощью крана 4 сообщается с сосудом, в котором измеряют давление. При увеличении давления трубка разгибается. Движение её закрытого конца при помощи рычага 5 и зубчатки 3 передается стрелке 2, движущейся около шкалы прибора. При уменьшении давления трубка, благодаря своей упругости, возвращается в прежнее положение, а стрелка — к нулевому делению шкалы.

Поршневой жидкостный насос.

В опыте, рассмотренном нами ранее (§ 40), было установлено, что вода в стеклянной трубке под действием атмосферного давления поднималась вверх за поршнем. На этом основано действие поршневых насосов.

Насос схематически изображен на рисунке. Он состоит из цилиндра, внутри которого ходит вверх и вниз, плотно прилегая к стенкам сосуда, поршень 1. В нижней части цилиндра и в самом поршне установлены клапаны 2, открывающиеся только вверх. При движении поршня вверх вода под действием атмосферного давления входит в трубу, поднимает нижний клапан и движется за поршнем.

При движении поршня вниз вода, находящаяся под поршнем, давит на нижний клапан, и он закрывается. Одновременно под давлением воды открывается клапан внутри поршня, и вода переходит в пространство над поршнем. При следующем движении поршня вверх в месте с ним поднимается и находящаяся над ним вода, которая и выливается в отводящую трубу. Одновременно за поршнем поднимается и новая порция воды, которая при последующем опускании поршня окажется над ним, и вся эта процедура повторяется вновь и вновь, пока работает насос.

Гидравлический пресс.

Закон Паскаля позволяет объяснить действие гидравлической машины (от греч. гидравликос — водяной). Это машины, действие которых основано на законах движения и равновесия жидкостей.

Основной частью гидравлической машины служат два цилиндра разного диаметра, снабженные поршнями и соединительной трубкой. Пространство под поршнями и трубку заполняют жидкостью (обычно минеральным маслом). Высоты столбов жидкости в обоих цилиндрах одинаковы, пока на поршни не действуют силы.

Допустим теперь, что силы F1 и F2 — силы, действующие на поршни, S1 и S2 — площади поршней. Давление под первым (малым) поршнем равно p1 = F1 / S1, а под вторым (большим) p2 = F2 / S2 . По закону Паскаля давление покоящейся жидкостью во все стороны передается одинаково, т. е. p1 = p2 или F1 / S1 = F2 / S2 , откуда:

F2 / F1 = S2 / S1 .

Следовательно, сила F2 во столько раз больше силы F1 , во сколько раз площадь большого поршня больше площади малого поршня. Например, если площадь большого поршня 500 см2, а малого 5 см2, и на малый поршень действует сила 100 Н, то на больший поршень будет действовать сила, в 100 раз бóльшая, то есть 10 000 Н.

Таким образом, с помощью гидравлической машины можно малой силой уравновесить бóльшую силу.

Отношение F1 / F2 показывает выигрыш в силе. Например, в приведенном примере выигрыш в силе равен 10 000 Н / 100 Н = 100.

Гидравлическая машина, служащая для прессования (сдавливания), называется гидравлическим прессом.

Гидравлические прессы применяются там, где требуется большая сила. Например, для выжимания масла из семян на маслобойных заводах, для прессования фанеры, картона, сена. На металлургических заводах гидравлические прессы используют для изготовления стальных валов машин, железнодорожных колес и многих других изделий. Современные гидравлические прессы могут развивать силу в десятки и сотни миллионов ньютонов.

Устройство гидравлического пресса схематически показано на рисунке. Прессуемое тело 1 (A) кладут на платформу, соединенную с большим поршнем 2 (B). При помощи малого поршня 3 (D) создается большое давление на жидкость. Это давление передается в каждую точку жидкости, заполняющей цилиндры. Поэтому такое же давление действует и на второй, большой поршень. Но так как площадь 2-го (большого) поршня больше площади малого, то и сила, действующая на него, будет больше силы, действующей на поршень 3 (D). Под действием этой силы поршень 2 (B) будет подниматься. При подъеме поршня 2 (B) тело (A) упирается в неподвижную верхнюю платформу и сжимается. При помощи манометра 4 (M) измеряется давление жидкости. Предохранительный клапан 5 (P) автоматически открывается, когда давление жидкости превышает допустимое значение.

Из малого цилиндра в большой жидкость перекачивается повторными движениями малого поршня 3 (D). Это осуществляется следующим образом. При подъеме малого поршня (D) клапан 6 (K) открывается, и в пространство, находящееся под поршнем, засасывается жидкость. При опускании малого поршня под действием давления жидкости клапан 6 (K) закрывается, а клапан 7 (K’) открывается, и жидкость переходит в большой сосуд.

Действие воды и газа на погруженное в них тело.

Под водой мы легко можем поднять камень, который с трудом поднимается в воздухе. Если погрузить пробку под воду и выпустить ее из рук, то она всплывет. Как можно объяснить эти явления?

Мы знаем (§ 38), что жидкость давит на дно и стенки сосуда. И если внутрь жидкости поместить какое-нибудь твердое тело, то оно также будет подвергаться давлению, как и стенки сосуда.

Рассмотрим силы, которые действуют со стороны жидкости на погруженное в нее тело. Чтобы легче было рассуждать, выберем тело, которое имеет форму параллелепипеда с основаниями, параллельными поверхности жидкости (рис.). Силы, действующие на боковые грани тела, попарно равны и уравновешивают друг друга. Под действием этих сил тело сжимается. А вот силы, действующие на верхнюю и нижнюю грани тела, неодинаковы. На верхнюю грань давит сверху силой F1 столб жидкости высотой h1 . На уровне нижней грани давление производит столб жидкости высотой h2. Это давление, как мы знаем (§ 37), передается внутри жидкости во все стороны. Следовательно, на нижнюю грань тела снизу вверх с силой F2 давит столб жидкости высотой h2. Но h2 больше h1, следовательно, и модуль силы F2 больше модуля силы F1. Поэтому тело выталкивается из жидкости с силой Fвыт, равной разности сил F2F1 , т. е.

Fвыт = F2F1

Рассчитаем эту выталкивающую силу. Силы F1 и F2 , действующие на верхнюю и нижнюю грани параллелепипеда, можно вычислить, зная площади этих граней (S1 и S2) и давление жидкости на уровнях этих граней (p1 и p2):

F1 = p1·S1, а F2 = p2·S2, так как p1 = ρж·g·h1 , p2 = ρж·g·h2 , а S1 = S2 = S, где S — площадь грани параллелепипеда (все грани равны).

Тогда, Fвыт = F2 — F1 = ρ·g·h2·S — ρ·g·h1·S = ρ·g·S·(h2 — h1) = ρ·g·S·h, где h — высота параллелепипеда (h = h2 — h1).

Но S·h = V, где V — объем параллелепипеда, а ρж·V = mж — масса жидкости в объеме параллелепипеда. Следовательно,

Fвыт = g·mж = Pж ,

т. е. выталкивающая сила равна весу жидкости в объеме погруженного в нее тела (выталкивающая сила равна весу жидкости такого же объёма, как и объём погруженного в нее тела).

Существование силы, выталкивающей тело из жидкости, легко обнаружить на опыте.

На рисунке а изображено тело, подвешенное к пружине со стрелкой-указателем на конце. Стрелка отмечает на штативе растяжение пружины. При отпускании тела в воду пружина сокращается (рис., б). Такое же сокращение пружины получится, если действовать на тело снизу вверх с некоторой силой, например, нажать рукой (приподнять).

Следовательно, опыт подтверждает, что на тело, находящееся в жидкости, действует сила, выталкивающая это тело из жидкости.

К газам, как мы знаем, также применим закон Паскаля. Поэтому на тела, находящиеся в газе, действует сила, выталкивающая их из газа. Под действием этой силы воздушные шары поднимаются вверх. Существование силы, выталкивающей тело из газа, можно также наблюдать на опыте.

К укороченной чашке весов подвесим стеклянный шар или большую колбу, закрытую пробкой. Весы уравновешиваются. Затем под колбу (или шар) ставят широкий сосуд так, чтобы он окружал всю колбу. Сосуд наполняется углекислым газом, плотность которого больше плотности воздуха (поэтому углекислый газ опускается вниз и заполняет сосуд, вытесняя из него воздух). При этом равновесие весов нарушается. Чашка с подвешенной колбой поднимается вверх (рис.). На колбу, погруженную в углекислый газ, действует бóльшая выталкивающая сила, по сравнению с той, которая действует на нее в воздухе.

Сила, выталкивающая тело из жидкости или газа, направлена противоположно силе тяжести, приложенной к этому телу.

Поэтому пролкосмосе). Именно этим объясняется, что в воде мы иногда легко поднимаем тела, которые с трудом удерживаем в воздухе.

Архимедова сила.

Силу, с которой тело, находящееся в жидкости, выталкивается ею, можно рассчитать (как это сделано в § 48). А можно определить ее значение на опыте, используя для этого прибор, изображенный на рисунке.

К пружине подвешивается небольшое ведерко и тело цилиндрической формы (рис., а). Стрелка на штативе отмечает растяжение пружины. Она показывает вес тела в воздухе. Приподняв тело, под него подставляется отливной сосуд, наполненный жидкостью до уровня отливной трубки. После чего тело погружается целиком в жидкость (рис., б). При этом часть жидкости, объем которой равен объему тела, выливается из отливного сосуда в стакан. Пружина сокращается, и указатель пружины поднимается вверх, показывая уменьшение веса тела в жидкости. В данном случае на тело, кроме силы тяжести, действует еще одна сила, выталкивающая его из жидкости. Если в верхнее ведерко вылить жидкость из стакана (т. е. ту, которую вытеснило тело), то указатель пружины возвратится к своему начальному положению (рис., в).

На основании этого опыта можно заключить, что сила, выталкивающая целиком погруженное в жидкость тело, равна весу жидкости в объеме этого тела. Такой же вывод мы получили и в § 48.

Если подобный опыт проделать с телом, погруженным в какой-либо газ, то он показал бы, что сила, выталкивающая тело из газа, также равна весу газа, взятого в объеме тела.

Сила, выталкивающая тело из жидкости или газа, называется архимедовой силой, в честь ученого Архимеда, который впервые указал на ее существование и рассчитал ее значение.

Итак, опыт подтвердил, что архимедова (или выталкивающая) сила равна весу жидкости в объеме тела, т. е. FА = Pж = g·mж. Массу жидкости mж, вытесняемую телом, можно выразить через ее плотность ρж и объем тела Vт, погруженного в жидкость (так как Vж — объем вытесненной телом жидкости равен Vт — объему тела, погруженного в жидкость), т. е. mж = ρж·Vт. Тогда получим:

FA = g·ρж·Vт

Следовательно, архимедова сила зависит от плотности жидкости, в которую погружено тело, и от объема этого тела. Но она не зависит, например, от плотности вещества тела, погружаемого в жидкость, так как эта величина не входит в полученную формулу.

Определим теперь вес тела, погруженного в жидкость (или в газ). Так как две силы, действующие на тело в этом случае, направлены в противоположные стороны (сила тяжести вниз, а архимедова сила вверх), то вес тела в жидкости P1 будет меньше веса тела в вакууме P = g·m на архимедову силу FА = g·mж (где mж — масса жидкости или газа, вытесненной телом).

Таким образом, если тело погружено в жидкость или газ, то оно теряет в своем весе столько, сколько весит вытесненная им жидкость или газ.

Пример. Определить выталкивающую силу, действующую на камень объемом 1,6 м3 в морской воде.

Запишем условие задачи и решим ее.

Дано:

Vт =1,6 м3

ρж = 1030 кг/м3

g = 9,8 Н/кг

FА — ?

Решение:

FА = g·ρж·Vт,

FА = 9.8 Н/кг · 1030 кг/м3 · 1,6 м3 = 16 480 Н ≈ 16,5 кН.

Ответ: FА = 16,5 кН.

Плавание тел.

На тело, находящееся внутри жидкости, действуют две силы: сила тяжести, направленная вертикально вниз, и архимедова сила, направленная вертикально вверх. Рассмотрим, что будет происходить с телом под действием этих сил, если в начале оно было неподвижно. При этом возможны три случая:

1) если сила тяжести Fтяж больше архимедовой силы FА, то тело будет опускаться на дно, тонуть, т. е. если

Fтяж > FА, то тело тонет;

2) если сила тяжести равна архимедовой силе, то тело может находиться в равновесии в любом месте жидкости, т. е. если

Fтяж = FА, то тело плавает;

3) если сила тяжести меньше архимедовой силы, то тело будет подниматься из жидкости, всплывать, т. е. если

Fтяж < FА, то тело всплывает.

Рассмотрим последний случай подробнее.

Когда всплывающее тело достигнет поверхности жидкости, то при дальнейшем его движении вверх архимедова сила будет уменьшаться. Почему? А потому, что будет уменьшаться объем части тела, погруженной в жидкость, а архимедова сила равна весу жидкости в объеме погруженной в нее части тела.

Когда архимедова сила станет равной силе тяжести, тело остановится и будет плавать на поверхности жидкости, частично погрузившись в нее.

Полученный вывод легко проверить на опыте.

В отливной сосуд нальем воду до уровня отливной трубки. После этого погрузим в сосуд плавающее тело, предварительно взвесив его в воздухе. Опустившись в воду, тело вытесняет объем воды, равный объему погруженной в нее части тела. Взвесив эту воду, находим, что ее вес (архимедова сила) равен силе тяжести, действующей на плавающее тело, или весу этого тела в воздухе.

Проделав такие же опыты с любыми другими телами, плавающими в разных жидкостях — в воде, спирте, растворе соли, можно убедиться, что если тело плавает в жидкости, то вес вытесненной им жидкости равен весу этого тела в воздухе.

Легко доказать, что если плотность сплошного твердого тела больше плотности жидкости, то тело в такой жидкости тонет. Тело с меньшей плотностью всплывает в этой жидкости. Кусок железа, например, тонет в воде, но всплывает в ртути. Тело же, плотность которого равна плотности жидкости, остается в равновесии внутри жидкости.

Плавает на поверхности воды лед, так как его плотность меньше плотности воды.

Чем меньше плотность тела по сравнению с плотностью жидкости, тем меньшая часть тела погружена в жидкость.

При равных плотностях тела и жидкости тело плавает внутри жидкости на любой глубине.

Две несмешивающиеся жидкости, например вода и керосин, располагаются в сосуде в соответствии со своими плотностями: в нижней части сосуда — более плотная вода (ρ = 1000 кг/м3), сверху — более легкий керосин (ρ = 800 кг/м3).

Средняя плотность живых организмов, населяющих водную среду, мало отличается от плотности воды, поэтому их вес почти полностью уравновешивается архимедовой силой. Благодаря этому водные животные не нуждаются в столь прочных и массивных скелетах, как наземные. По этой же причине эластичны стволы водных растений.

Плавательный пузырь рыбы легко меняет свой объем. Когда рыба с помощью мышц опускается на большую глубину, и давление воды на нее увеличивается, пузырь сжимается, объем тела рыбы уменьшается, и она не выталкивается вверх, а плавает в глубине. Таким образом, рыба может в определенных пределах регулировать глубину своего погружения. Киты регулируют глубину своего погружения за счет уменьшения и увеличения объема легких.

Плавание судов.

Суда, плавающие по рекам, озерам, морям и океанам, построены из разных материалов с различной плотностью. Корпус судов обычно делается из стальных листов. Все внутренние крепления, придающие судам прочность, также изготовляют из металлов. Для постройки судов используют различные материалы, имеющие по сравнению с водой как бóльшие, так и меньшие плотности.

Благодаря чему суда держатся на воде, принимают на борт и перевозят большие грузы?

Опыт с плавающим телом (§ 50) показал, что тело вытесняет своей подводной частью столько воды, что по весу эта вода равна весу тела в воздухе. Это также справедливо и для любого судна.

Вес воды, вытесняемой подводной частью судна, равен весу судна с грузом в воздухе или силе тяжести, действующей на судно с грузом.

Глубина, на которую судно погружается в воду, называется осадкой. Наибольшая допускаемая осадка отмечена на корпусе судна красной линией, называемой ватерлинией (от голланд. ватер — вода).

Вес воды, вытесняемой судном при погружении до ватерлинии, равный силе тяжести, действующей на судно с грузом, называется водоизмещением судна.

В настоящее время для перевозки нефти строятся суда водоизмещением 5 000 000 кН (5 · 106 кН) и больше, т. е. имеющие вместе с грузом массу 500 000 т (5 · 105 т) и более.

Если из водоизмещения вычесть вес самого судна, то мы получим грузоподъемность этого судна. Грузоподъемность показывает вес груза, перевозимого судном.

Судостроение существовало еще в Древнем Египте, в Финикии (считается, что Финикийцы были одними из лучших судостроителей), Древнем Китае.

В России судостроение зародилось на рубеже 17-18 вв. Сооружались главным образом военные корабли, но именно в России были построены первый ледокол, суда с двигателем внутреннего сгорания, атомный ледокол «Арктика».

Воздухоплавание.

Рисунок с описанием шара братьев Монгольфье 1783 года: «Вид и точные размеры „Аэростата Земной шар“, который был первым». 1786

С давних времен люди мечтали о возможности летать над облаками, плавать в воздушном океане, как они плавали по морю. Для воздухоплавания

вначале использовали воздушные шары, которые наполняли или нагретым воздухом, или водородом либо гелием.

Для того, чтобы воздушный шар поднялся в воздух, необходимо, чтобы архимедова сила (выталкивающая) FА, действующая на шар, была больше силы тяжести Fтяж, т. е. FА > Fтяж.

По мере поднятия шара вверх архимедова сила, действующая на него, уменьшается (FА = gρV), так как плотность верхних слоев атмосферы меньше, чем у поверхности Земли. Чтобы подняться выше, с шара сбрасывается специальный балласт (груз) и этим облегчает шар. В конце концов шар достигает своей своей предельной высоты подъема. Для спуска шара из его оболочки при помощи специального клапана выпускается часть газа.

В горизонтальном направлении воздушный шар перемещается только под действием ветра, поэтому он называется аэростатом (от греч аэр — воздух, стато — стоящий). Для исследования верхних слоев атмосферы, стратосферы еще не так давно применялись огромные воздушные шары — стратостаты.

До того как научились строить большие самолеты для перевозки по воздуху пассажиров и грузов, применялись управляемые аэростаты — дирижабли. Они имеют удлиненную форму, под корпусом подвешивается гондола с двигателем, который приводит в движение пропеллер.

Воздушный шар не только сам поднимается вверх, но может поднять и некоторый груз: кабину, людей, приборы. Поэтому для того, чтобы узнать, какой груз может поднять воздушный шар, необходимо определить его подъемную силу.

Пусть, например, в воздух запущен шар объемом 40 м3, наполненный гелием. Масса гелия, заполняющая оболочку шара, будет равна:
mГе = ρГе·V = 0,1890 кг/м3 · 40 м3 = 7,2 кг,
а его вес равен:
PГе = g·mГе ; PГе = 9,8 Н/кг · 7,2 кг = 71 Н.
Выталкивающая же сила (архимедова), действующая на этот шар в воздухе, равна весу воздуха объемом 40 м3, т. е.
FА = g·ρвоздV; FА = 9,8 Н/кг · 1,3 кг/м3 · 40 м3 = 520 Н.

Значит, этот шар может поднять груз весом 520 Н — 71 Н = 449 Н. Это и есть его подъемная сила.

Шар такого же объема, но наполненный водородом, может поднять груз 479 Н. Значит, подъемная сила его больше, чем шара, наполненного гелием. Но все же чаще используют гелий, так как он не горит и поэтому безопаснее. Водород же горючий газ.

Гораздо проще осуществить подъем и спуск шара, наполненного горячим воздухом. Для этого под отверстием, находящимся в нижней части шара, располагается горелка. При помощи газовой горелки можно регулировать температуру воздуха внутри шара, а значит, его плотность и выталкивающую силу. Чтобы шар поднялся выше, достаточно сильнее нагреть воздух в нем, увеличив пламя горелки. При уменьшении пламени горелки температура воздуха в шаре уменьшается, и шар опускается вниз.

Можно подобрать такую температуру шара, при которой вес шара и кабины будет равен выталкивающей силе. Тогда шар повиснет в воздухе, и с него будет легко проводить наблюдения.

По мере развития науки происходили и существенные изменения в воздухоплавательной технике. Появилась возможность использования новых оболочек для аэростатов, которые стали прочными, морозоустойчивыми и легкими.

Достижения в области радиотехники, электроники, автоматики позволили сконструировать беспилотные аэростаты. Эти аэростаты используются для изучения воздушных течений, для географических и медико-биологических исследований в нижних слоях атмосферы.

Ссылки

  • Уроки по физике за 7 класс по школьной программе

План урока:

Можно ли бежать по снегу?

Способы изменения давления

Давление газа

Закон Паскаля

Давление жидкости

Сообщающиеся сосуды

Можно ли бежать по снегу?

В соревнованиях, когда одна команда бежит на лыжах по рыхлому снегу, а другая без лыж победит, конечно же, команда с лыжами. Все дело в лыжах. Когда человек стоит на лыжах, его вес равномерно распределяется по всей площади лыж. Нетрудно сравнить площадь двух лыж и площадь подошв обуви.

1
                                                                 

Площадь лыж больше площади подошв. Если одинаковый вес человека распределить на разные площади, получится, что каждой единице площади лыж достанется веса меньше, чем такой же единице площади подошв. (Чтобы лучше понять это, надо представить простую ситуацию: на разные по размерам куски хлеба намазать по одинаковому количеству сливочного масла. На маленьком куске хлеба получится толстый слой масла, а на большом — масло размажется очень тонко).

Сила, которая давит на длинные лыжи и небольшие подошвы – это вес человека. А вот сила, которая приходится на единицу площади лыжи или подошвы – это уже не полный вес человека, а лишь маленькая часть этого веса. Эту силу в физике называют давлением.

Значит, команда на лыжах выиграла, так как оказывала на снег меньшее давление, и передвигаться ей, не проваливаясь, было легче.

Нужно отличать давление от силы давления. В примере в роли силы давления выступает вес человека, а давление – это часть веса, которая достается единице площади.

Давление принято обозначать буквой p (маленькая латинская буква) и находить по правилу:

p = F/S.

Достаточно понятная формула: «силу давления F разделить (распределить) на площадь S».

2
Источник

Давление измеряют и другими единицами:

3
Источник

Давление оказывают тела не только своим весом. Когда, например, режут хлеб или сыр на части, то нож разрезает продукты под действием мышечной силы руки.

Зная давление, можно найти силу, которая действует на поверхность:

F = p ∙ S

Способы изменения давления

Жарким летним днем, когда асфальт будто плавится, на нем остаются следы.

Какая обувь оставляет самые заметные, глубокие следы? Несомненно, ответ прост: это шпильки женских туфель. Маленькая площадь, следовательно, большое давление – и видимый результат.

4
 

Иногда давление необходимо сделать большим, иногда — маленьким.

Большое давление нужно там, где имеют дело с режущими и колющими инструментами. Необходимо, чтобы они были остро заточены, чтобы площадь соприкосновения была минимальной. Если нажать посильнее, то можно получить огромные давления. Резец токарного станка срезает стружку, оказывая давление на деталь 2,45 ∙ 109 Па.

5
Источник

В рукопашном бое востока преобладают колющие и рубящие удары пальцами рук, ног и ребром ладони. Обладая маленькой массой, можно нанести резкий болевой удар, ведь площадь кончиков пальцев или ребра ладони значительно меньше площади всей ладони или кулака. А значит, при одинаковой силе можно оказать большее давление на место удара.

А какое давление оказывает лезвие из дамасской стали, если легкая шелковая лента, упав на саблю, перерезается пополам!

В тех случаях, когда давление должно быть маленьким, сила должна быть тоже небольшой, а это в современных технологиях встречается редко. Тогда для уменьшения давления нужно увеличить площадь опоры.

Гусеницы тракторов, вездеходов значительно уменьшают давление на почву (всего 40 — 50 кПа), поэтому этот вид транспорта хорошо проходим даже в условиях болота. Для сравнения давление, которое оказывают колеса легкового автомобиля на дорогу равно 200 – 300 кПа, причем вес автомобиля значительно меньше, чем у трактора. Человек для прохождения по болотистой почве применяет специальные болотоходы, а по снегу – снегоходы.

6
 

Для того, чтобы дома и другие строения не давали осадку, необходимо под них подводить широкий фундамент (увеличение площади). Известная всему миру Останкинская башня, высотой 533 м, давит своим фундаментом на землю всего в 270 кПа (как легковой автомобиль). Таким же фундаментом для железнодорожных рельсов являются шпалы, уменьшающие давление вагонов на рельсы.

Для похода лучше выбирать рюкзак с широкими лямками, даже не нужно объяснять, почему.

Животным природа подарила возможность учитывать и использовать давление:

  • клыки (хищники легко разрывают ими мясо);
  • когти (кошки свободно забираются на дерево, спасаясь от собаки);
  • острые зубы (пираньи – хищные рыбы пресных вод в считанные секунды обгладывают свои жертвы):
  • клювы (дятел быстро долбит кору деревьев, находя там жучков для еды);
  • острые носы (комар мгновенно протыкает кожу людей и животных и высасывает кровь);

7
                                                        

Приведенные примеры говорят о том, как животные приспособились к борьбе за существование.

Итак, давление учитывает и человек, и природа, стремясь держать его большим или маленьким в нужных случаях.

Давление газа

Воздушные шары и мыльные пузыри знакомы всем взрослым и детям. Это обязательный атрибут игр или праздников. А как получить такой шарик? Он наполняется воздухом, который человек вдувает в шарик при выдохе. С каждым выдохом шарик растет все больше, и делается все более упругим. На резину шарика воздух давит, стараясь растянуть его в разные стороны. Силе давления противостоит сила упругости деформированного шарика, и нужно вовремя остановиться, иначе сила давления перерастет силу упругости, произойдет громкий хлопок – шарик лопнет.

8
 

Твердое тело весом давит на часть поверхности, на которой стоит (если ведро стоит на полу, то оно и давит только на пол), а газ действует во всех направлениях одинаково, поэтому мыльные пузыри имеют форму шара. Форму воздушного шара определяет резина, из которой он изготовлен. Если резина везде одинакова, то шар будет круглым. Если резина имеет в разных частях шара разную упругость, то и шар надуется по-разному, и будет иметь заданную форму.

Молекулы газообразного вещества мечутся беспорядочно и хаотично, но такой хаос большого количества частиц обеспечивает некий порядок: в любых направлениях зигзагообразными траекториями движется примерно одинаковое количество молекул. Суммарная сила удара сразу всех молекул, действующая на единицу площади, и образует давление газа.

9
Источник

Если накачивать насосом камеру велосипеда или мяча, то с каждым качком камера становится будто бы более твердой, потому что молекулы воздуха все прибывают и прибывают, сила суммарного удара по стенкам камеры становится все больше и больше. А значит, давление растет. Если воздух выходит, то камера на глазах «худеет», давление уменьшается. С ростом числа молекул связано увеличение массы газа, а уменьшение числа молекул ведет к уменьшению массы. Поэтому давление газа зависит от его массы, которую опытным путем гораздо легче определить, чем само давление.

10
 

С повышением температуры тела молекулы перемещаются быстрее. Чем выше скорость, тем сильнее молекулы ударяются о сосуд, в котором газ находится. Значит, чем температура выше, тем значительнее давление, оказываемое газом.

Еще интересный пример изменения давления. Надо взять бумажный пакет, надуть его и резко ударить по нему ладонью. Хлопок, и пакет разорван. Все очень быстро и просто, а ведь здесь произошли вполне объяснимые физические процессы. Кулек под ударом ладони сжимается, объем газа в нем становится меньше, количество молекул не увеличивается, но так как молекулы расположены плотнее, на единицу площади стенок их количество растет, что приводит к росту давления. Давление разрывает бумагу, резко вырвавшийся воздух является причиной звука в виде хлопка. Получается, давление газа связано и с объемом: давление становится больше с уменьшением объема газа, а если объем газа увеличивается, то давление его уменьшается.

И следующее. У разных газов молекулы различны по массе. С ростом молекулярной массы увеличивается ударная сила молекулы о поверхность, что приводит к росту давления. Поэтому различные газы в одном объеме, с одной и той же температурой оказывают неодинаковые давления.

Итак:

  • давление газа определяется общими ударами большого числа молекул о поверхности, окружающие газ;
  • если изменить количество молекул газа, его температуру или объем сосуда, в котором газ находится, то можно изменить давление этого газа;
  • в одинаковых прочих условиях (число молекул, объем, температура) у различных газов давление неодинаково.

Закон Паскаля

Твердые тела, соприкасаясь с поверхностью, давят на нее перпендикулярно. Газы давят в различных направлениях. А как давят жидкости?

Молекулы жидкости, перемещаясь, долго не задерживаются на месте. Здесь они подобны молекулам газов. Различие лишь состоит в расстояниях между молекулами. Поэтому в жидкостях, как и газах, есть давление, которое действует в любых направлениях одинаково. Почему в ванной вода из крана течет одной струей, а эта же самая вода из душа течет большим количеством одинаковых струек? Если бы вместо душевого сита был шар с дырками, то вода лилась бы во все стороны, и даже вверх. Объясняет эти явления знаменитый закон Паскаля:

11
Источник

Закон Паскаля справедлив для газов тоже. Примером из жизни служит уже рассмотренное надувание воздушного шарика. Струю воздуха человек направляет прямо перед собой (он же не крутит головой, чтобы заполнить шар), а шар заполняется везде, во всех направлениях. Накачивая резиновую лодку ручным насосом, рыбак нажимает сверху вниз на насос, воздух же наполняет полностью всю форму лодки.

Этот закон давления нашел широчайшее применение в механизмах, использующих принцип работы гидравлического пресса (изобретен Б. Паскалем). Если нажать на маленький поршень S1 силой F1, давление получится p1 = F1/ S1. Это давление без изменения (по закону Паскаля) будет передаваться во всех направлениях и дойдет до поршня S2. Значит, там давление p2 = F2/S2 такое же. p1 = p2, то есть F1/ S1 = F2/S2.

Используя математику, из этого равенства можно получить следующее: F1/F2 = S1/S2. Это равенство выражает закон гидравлического пресса (гидравлический — водяной или жидкостный). Оно означает: действуя маленькой силой на небольшую площадь можно получить большую силу, действующую на большую площадь. Силы будут отличаться во столько же раз, во сколько отличаются площади. Если действовать на малую площадь маленькой силой, используя площадь в несколько раз большую, можно получить силу во столько же раз большую.

12
Источник

Само слово «пресс» обозначает: механизм для обработки материалов давлением.

С помощью пресса макулатуре, металлолому, сену придают формы удобные для транспортировки.

Методом прессования изготовляют детали сложнейших приборов и машин.

В водяных насосах, отбойных молотках, также используется закон Паскаля и закон гидравлического пресса. Нажимая на тормоз, шофер останавливает автомобиль. Усиленное давление по этим законам передается на тормозные колодки, которые не дают вращаться колесам. Но это, конечно, далеко не весь перечень примеров проявления и использования свойств давления.

Давление жидкости

Можно ли численно рассчитать давление жидкости? Ответ получается, если применить нетрудные математические преобразования.

По определению p = F/S. F – сила, с которой жидкость (ведь она обладает весом P) давит на дно. Дальше, вспомнив формулы веса (P = mg), массы (m = ρV), объема (V = Sh), можно получить:

p = P/S = mg/S = ρVg/S = ρShg/S. Сократить полученную дробь на S, и результат готов:

p = ρgh

Изначально в формуле была задействована площадь дна, а в результате получилось выражение, в котором давление жидкости зависит ни от формы сосуда, ни от площади его стенок и дна, а от плотности ρ и глубины h жидкости (физики называют глубину высотой столба жидкости). Число g ≈ 10 Н/кг осталось от формулы веса.

13 Источник                                                                  Источник

А теперь самое интересное. Можно ли водой из кружки разрушить большую бочку? Тот факт, что высота столба определяет давление жидкости, позволяет ответить на этот вопрос: « Да, можно!»

Если взять несколько сосудов различной формы, одинаковой высоты и с одинаковой площадью дна, которое затянуто эластичной пленкой, наполнить сосуды водой, то дно прогнется одинаково во всех трех случаях. Хотя вес жидкости разный, сила давления возникает одинаковая (но ведь силой давления является вес!). Что происходит? А вот что:

F = p ∙ S, а p = ρgh и F =  ρgh ∙ S

Одна жидкость, плотность ρ, высота h и площадь S дна — одинаковы. Значит, и сила давления одинаковая. Это явление «гидростатического парадокса». 

14
Источник

Первое объяснение этому парадоксу (противоречие с общепринятым утверждением; перевод с греческого «докса» — «мнение», «пара» — «против») дано опять же Б.Паскалем.

15Источник

Явления, связанные со стоячей жидкостью (в сосуде, в луже, в пруду) и давлением называют гидростатическими.

Сообщающиеся сосуды

Красивое зрелище – действующие фонтаны. Они всегда привлекают к себе внимание детей и взрослых. А каков же их принцип действия?

Первый человек, который это объяснил, опять был Блез Паскаль. Если взять две стеклянные трубочки и соединить их трубкой резиновой, и через воронку наливать воду в одну трубочку, то при любом положении трубочек вода в них будет на одинаковом уровне.

Эти трубочки, а на их месте могут быть любые сосуды, соединяются (сообщаются) друг с другом. Давление в обеих трубочках одинаково (по закону Паскаля), значит, и высота столбиков жидкости в этих соединенных, сообщающихся сосудах — одинакова.

16
Источник

Если один сосудов поднять на большую высоту, то из второго сосуда жидкость будет стремиться подняться до уровня первого сосуда, но сила тяжести, возвращает ее обратно на землю. Таков основной принцип работы фонтана. Вот его примерная схема:

17

Фонтан соединяется с большой башней, в которую закачивается вода с помощью насосов. Такую башню называют водонапорной. А если водонапорную башню соединить с водопроводной сетью, то вода поступит в любую квартиру, в любое здание. Водопровод — это пример применения в жизни сообщающихся сосудов.

18
Источник

К сообщающимся сосудам относятся насосы со шлангами, лейки, чайники, кофейники. В них по два сосуда: емкость под жидкость и носик. Звучит непривычно, но это так.

При помощи сообщающихся сосудов измеряют небольшое давление. Прибор называется жидкостным манометром. Один сосуд закрыт, а ко второму эластичной трубкой присоединена пластмассовая коробочка, затянутая пленкой. Если на пленку нажать, по закону Паскаля давление передастся на жидкость. В одном колене такого манометра жидкость опускается, а в другом поднимается. Давление тем больше, чем больше разница уровней жидкости.

19 Источник

Принцип сообщающихся сосудов используется в шлюзах – устройствах, пропускающих речные корабли через плотины, перегораживающие реки. Плотины обычно устраивают там, где работают гидроэлектростанции. Плотины разделяют течение реки на части: верхнее течение (там накапливается вода) и нижнее. Там, где проходят корабли, сделаны специальные огромные ворота, которые удерживают воду. Получается, что большие сообщающиеся сосуды между собой перекрыты.

Пусть корабль идет по течению реки вниз. Он попадает в верхнюю часть шлюза, где уровень воды высокий. Ворота открываются, и вода начинает перетекать в нижнюю часть, уровень воды снижается, а вместе с ним опускается и корабль (в нижней части наоборот уровень повышается). Снижение происходит до тех пор, пока не произойдет выравнивание (закон сообщающихся сосудов) верхнего и нижнего течений. Корабль может дальше двигаться по реке. Если корабль движется вверх по течению, он вначале попадает в нижнюю часть шлюза. Ворота открываются, вода из верхней части поступает в нижнюю до выравнивания уровней. В нижней части вода поднимается, и вместе с ней поднимается корабль. После выравнивания уровней корабль может двигаться дальше. Шлюзы очень больших плотин могут иметь не одни ворота, а сразу несколько. Ворота могут открываться, как обычные двери (по течению реки) или вертикально.

20
Источник

Если дно непрозрачного бака соединить с тонкой стеклянной трубкой снизу, расположенной вертикально ему, то можно увидеть высоту воды в баке (см. рисунок). Так устроены водомерные стекла в паровых котлах.

Важный момент. В данных примерах используется одна жидкость. А если в разные сосуды налить разные жидкости, например, воду и керосин. Что будет наблюдаться?

Давление в обоих сосудах одинаково (закон Паскаля). Но ведь плотность воды 1000 кг/м3, а керосина – 800 кг/м3, тогда столб керосина с меньшей плотностью должен быть выше, чем столб воды в 1,25 раза, т. к. плотности отличаются тоже в 1,25 раза.

Наблюдается обратно пропорциональная зависимость:

h1/h2 = ρ21

21Источник

Значит, в сообщающихся сосудах уровни одинаковы для однородной жидкости, а для разных жидкостей уровни различны.

  • Определение давления в физике
    • Общая формула давления
    • Единицы давления

  • Формула гидростатического давления
  • Парциальное давление и его формула
  • Формула давления идеального газа
  • Приборы для измерения давления
  • Рекомендованная литература и полезные ссылки
  • Расчет давления жидкости на дно и стенки сосуда, видео
  • Давление – очень важная физическая величина, играющая огромную роль, как в окружающей природе, так и жизни человека. Внешне незаметное человеческому глазу давление может очень хорошо ощущаться каждым из нас. Особенно хорошо это усвоили люди в возрасте, часто страдающие от повышенного давления (или наоборот от пониженного). Но в нашей статье мы больше поговорим именно о давлении в физике, о том, как оно измеряется и рассчитывается, какие есть формулы для расчетов давления разных субстанций: воздуха, жидкости или твердого тела.

    Определение давления в физике

    Под давлением в физике понимается термодинамическая величина, выраженная соотношением перпендикулярной силы давления на площадь поверхности, на которую она воздействует. При этом согласно закону Паскаля если система находится в состоянии равновесия, то давление на нее будет одинаковым для всех точек системы.

    В физике, как впрочем и химии, давление обозначают большой буквой Р, идущей от латинского слова «pressura» – давление. (В английском языке давление так и осталось почти без изменения – pressure).

    Общая формула давления

    Из классического определения того, что такое давление можно вывести общую формулу для его расчета. Выглядеть она будет таким образом:

    P = F/S

    Где F – это сила давления, а S – площадь поверхности на которую она действует. То есть иными словами формула нахождения давления – это сила, воздействующая на определенную поверхность, разделенная на площадь этой самой поверхности.

    Как видно из формулы, при расчете давления всегда действует следующий принцип: чем меньше пространство, на которое влияет сила, тем большее количество давящей силы на него приходится и наоборот.

    Это можно проиллюстрировать простым жизненным примером: хлеб легче всего порезать острым ножом, потому что у острого ножа заточенное лезвие, то есть площадь поверхности S из формулы у него минимальна, а значит, давление ножа на хлеб будет максимально равно приложенной силе F того кто держит нож. А вот тупым ножом порезать хлеб уже сложнее, так как у его лезвия большая площадь поверхности S, и давление ножа на хлеб будет меньшим, и значит, чтобы отрезать себе кусок хлеба нужно приложить большее количество силы F.

    Резать хлеб

    Общая формула давления, по сути, отлично описывает формулу давления твердого тела.

    Барометрическое

    Этим термином именуется давление атмосферы (гравитации) на все предметы и объекты, находящие в ней, включая непосредственно поверхность Земли. Большинству оно также известно под именем атмосферного.

    Его причисляют к термодинамическим параметрам, а его величина меняется относительно места и времени измерения, а также погодных условий и нахождения над/ниже уровня моря .

    Величина барометрического давления равна модулю силы атмосферы на площади единицу по нормали к ней.

    В стабильной атмосфере величина данного физического явления равна весу столпа воздуха на основание с площадью, равной единице.

    Норма барометрического давления — 101 325 Па (760 мм рт. ст. при 0 градусов Цельсия). При этом чем выше объект оказывается от поверхности Земли, тем более низким становится давление на него воздуха. Через каждый 8 км оно снижается на 100 Па.

    гидростатическое давление формула

    Благодаря этому свойству в горах вода в чайниках закивает намного быстрее, чем дома на плите. Дело в том, что давление влияет на температуру кипения: с его снижением последняя уменьшается. И наоборот. На этом свойстве построена работа таких кухонных приборов , как скороварка и автоклав. Повышение давления внутри их способствуют формированию в посудинах более высоких температур, нежели в обычных кастрюлях на плите.

    как находить давление формула

    Используется для вычисления атмосферного давления формула барометрической высоты. Выглядит она таким образом, как на фото ниже.

    парциальное давление формула

    Р – это искомая величина на высоте, Р0 – плотность воздуха возле поверхности, g – свободного падения ускорение, h – высота над Землей, м – молярная масса газа, т – температура системы, r – универсальная газовая постоянная 8,3144598 Дж⁄(моль х К), а е – это число Эйклера, равное 2.71828.

    Часто в представленной выше формуле давления атмосферного вместо R используется К – постоянная Больцмана. Через ее произведение на число Авогадро нередко выражается универсальная газовая постоянная. Она более удобна для расчетов, когда число частиц задано в молях.

    При проведении вычислений всегда стоит брать во внимание возможность изменения температуры воздуха из-за смены метеорологической ситуации или при наборе высоты над уровнем моря, а также географическую широту.

    давление воздуха формула

    Единицы давления

    Согласно стандартам Международной метрической системы давление измеряется в паскалях. Один паскаль из классической формулы равен одному Ньютону (Как мы знаем, Ньютон у нас единица измерения силы) разделенному на один квадратный метр.

    Но увы на практике паскаль оказывается очень маленькой единицей и использовать его для измерения давления не всегда удобно, поэтому часто для измерения давления применяют другие единицы:

  • Бары – один бар равен 105 паскалей
  • Миллиметры водяного столпа
  • Метры водяного столпа
  • Технические и физические атмосферы
  • Формула гидростатического давления

    Как мы знаем, разные агрегатные состояния вещества, имеют разные физические свойства. Жидкости своими свойствами отличаются от твердых тел, а газы в свою очередь отличаются от них всех. Поэтому вполне логично, что способы определения давления для жидкостей, твердых тел и газов также будут разными. Так, например, формула давления воды (или гидростатического давления) будет иметь следующий вид:

    P = p*g*h

    Где маленькая p – плотность вещества, g – ускорение свободного падения, h – высота.

    В частности эта формула объясняет, почему при погружении водолазов (или батискафа или подводной лодки) на глубину все больше возрастает давление окружающей воды. Также из этой формулы понятно, почему на предмет, погруженный в какой-нибудь кисель, будет воздействовать большее давление, чем на предмет, погруженный просто в воду, так как плотность киселя (p) выше, чем у воды, а чем выше плотность жидкости, тем выше ее гидростатическое давление.

    Приведенная нами формула гидростатического давления справедлива не только для жидкостей, но и для газов. Поэтому поднимаясь высоко в горы (где воздух более разрежен, а значит меньшее давление), как и спускаясь в подводные глубины, человек, водолаз или альпинист должен пройти специальную адаптацию, привыкнуть к тому, что на него будет воздействовать другое давление.

    Резкая смена давления может привести к кессоной болезни (в случае с водолазами) или к «горной» болезни (в случае с альпинистами). И «кесонка» и «горняшка», как их сленгово называют водолазы и альпинисты, вызвана резкой сменной давления окружающей среды. То есть, если не подготовленный человек начнет вдруг подниматься на Эверест, то он быстро словит «горняшку», а если этот же человек начнет опускаться на дно Мариинской впадины, то гарантировано получит «кесонку». В первом случае причиной будет не адаптация организма к пониженному давлению, а во втором – к повышенному.

    Декомпрессионая камера

    Американские водолазы в декомпрессионой камере, призванной подготовить их к глубоководным погружениям и адаптировать организм к высокому давлению океанских глубин.

    Принципы гидравлики

    Давление и поток

    Назначение давления и потока.

    При изучении основ гидравлики были использованы следующие термины: сила, передача энергии, работа и мощность. Эти термины используются при описании взаимоотношения давления и потока. Давление и поток — два основных параметра каждой гидравлической системы. Давление и поток взаимосвязаны, но выполняют разную работу.

    Давление сжимает или прикладывает усилие. Поток двигает предметы

    Водяной пистолет является хорошим примером давления и потока в применении. Нажатие на спусковой крючок создаёт давление внутри водяного пистолета. Вода под давлением вылетает из водяного пистолета и таким образом сбивает деревянного солдатика.

    Что такое давление?

    Давайте подумаем, как и почему создаётся давление. Текучая среда (газ и жидкость) стремится к расширению или происходит сопротивление при их сжатии. Это и есть давление.

    Когда вы накачиваете шину, вы создаёте в шине давление. Вы закачиваете в шину воздух больше и больше. Когда шина полностью наполнена воздухом, происходит нажатие на стенки шины. Такое нажатие является видом давления. Воздух является видом газа и может быть сжат.

    Сжатый воздух давит на стенки шины с одинаковой силой в каждой точке. Жидкость находится под давлением. Основное отличие состоит в том, что газы могут сжиматься в большей степени, чем жидкости.

    Одинаковая сила в каждой точке

    Давление в сжатой жидкости

    Если вы нажмёте на сжатую жидкость, возникнет давление. Так же как и в случае с шиной, давление одинаково в каждой точке бочки, содержащей жидкость. Если давление слишком велико, бочка может сломаться. Бочка сломается в слабом месте, а не там, где больше давление, потому что давление одинаково в каждой точке.

    Жидкость почти не сжимается

    Сжатая жидкость удобна при передаче силы по трубам, на изгибе, вверх, вниз, потому что жидкости почти несжимаемы и передача энергии происходит немедленно. Многие гидравлические системы используют масло. Это потому, что масло почти не сжимается. В тоже время, масло может использовать в качестве смазки.

    Закон Паскаля: Давление, производимое внешними силами на поверхность жидкости или газа, передаётся по всем направлениям без изменения.

    Отношение давление и силы

    По закону Паскаля, отношение между давлением и силой выражается формулами: P = F/A

    F = P х S, где P — давление, F — сила, S — площадь

    Гидравлический рычаг

    На модели поршня, показанной на рисунке ниже, можно увидеть пример уравновешивания различного веса через гидравлический рычаг. Паскаль открыл, как видно на этом примере, что малый вес малого поршня уравновешивает большой вес большого поршня, доказывая, что площадь поршня пропорционально весу. Это открытие применительно к сжимаемой жидкости. Причина, почему это возможно, это то, что жидкость всегда действует с равной силой на равную площадь.

    На рисунке изображён груз 2 кг и груз 100 кг. Площадь одного груза, весом 2 кг — 1 см2, давление составляет 2 кг/см2. Площадь другогогруза, весом 100 кг — 50 см2, давление составляет 2 кг/см2. Два веса уравновешивают друг друга.

    Механический рычаг

    Та же ситуация может быть проиллюстрирована на примере механического рычага на рисунке ниже.

    Кот весом 1 кг сидит на расстоянии 5 метров от центра тяжести рычага и уравновешивает кота весом 5 кг на расстоянии 1 метра от центра тяжести, подобно грузу на примере гидравлического рычага.

    Преобразование энергии гидравлического рычага

    Важно помнить, что жидкость действует равной силой на равную площадь. При работе это очень сильно помогает.

    Имеется два цилиндра одинакового размера. Когда мы нажимаем на один поршень с усилием 10 кг, другой поршень выдавливается с усилием 10 кг, потому что площадь каждого цилиндра одинаковая. Если площади разные, силы тоже разные.

    Например, допустим, что большой поршень имеет площадь 50 см2, а маленький поршень имеет площадь 1 см2, при усилии в 10 кг на маленький поршень происходит воздействие 10 кг/см2 на каждую часть большого клапана согласно закона Паскаля, поэтому большой поршень получает общую силу 500 кг. Мы используем давление для передачи энергии и выполнения работы.

    Имеется важный пункт при преобразовании энергии, а именно, отношение между силой и расстоянием. Вспомни, на механическом рычаге, малый вес требует длинный рычаг для достижения равновесия. Для того, чтобы поднять кота весом 5 кг на 10 см, кот весом 1 кг должен опустить рычаг на 50 см вниз.

    Давайте посмотрим на рисунок гидравлического рычага снова и подумаем о ходе малого поршня. Ход малого поршня 50 см необходим для передачи достаточного количества жидкости для передвижения поршня большого цилиндра на 1 см.

    Поток создаёт движение

    Что такое поток?

    При разнице давления в двух точках гидравлической системы, жидкость стремится к точке с наименьшим давлением. Такое движение жидкости называется потоком. Здесь приведены несколько примеров потока. Вода в городском водопроводе создаёт давление. Когда мы поворачиваем кран, то за счёт разности давления из крана течёт вода.

    В гидравлической системе поток создаёт насос. Насос создаёт непрерывный поток

    Скорость и величина потока

    Скорость и величина потока используются для измерения потока.

    Скорость показывает расстояние, пройденное за определённый промежуток времени.

    Величина потока показывает, сколько жидкости протекает через определённую точку за данный момент времени.

    Величина потока и скорость

    В гидравлическом цилиндре легко рассмотреть отношение между величиной потока и скоростью.

    Во первых, мы должны подумать об объёме цилиндра, который мы должны заполнить и затем подумать о ходе поршня.

    На рисунке показан цилиндр А длинной 2 метра и объёмом 10 литров и цилиндр В длинной 1 метр и объёмом 10 литров. Если закачать 10 литров жидкости в минуту в каждый цилиндр, полный ход обоих поршней длится 1 минуту. Поршень цилиндра А двигается в два раза быстрее, чем цилиндра В. Это происходит потому, что поршень должен пройти расстояние в два раза больше за один и тот же промежуток времени.

    Это значит, что цилиндр с меньшим диаметром двигается быстрее, чем цилиндр с большим диаметром при одинаковой скорости потока для обоих цилиндров. Если мы увеличим скорость потока до 20 л/мин, обе камеры цилиндра наполнятся в два раза быстрее. Скорость поршня должна увеличиться в два раза.

    Таким образом, мы имеем два пути увеличения скорости цилиндра. Один путём уменьшения размера цилиндра и другой за счёт увеличения скорости потока. Скорость цилиндра, таким образом, пропорциональна скорости потока и обратно пропорционально площади поршня.

    Давление и сила

    Создание давления

    Если вы надавите на пробку в бочке, заполненную жидкостью, пробка будет остановлена жидкостью. При нажатии, жидкость под давлением давит на стенки бочки. При чрезмерном нажатии возможен разрыв бочки.

    Путь наименьшего сопротивления

    Если имеется бочка с водой и отверстием. При нажатии на крышку сверху, вода вытекает из отверстия. Вода, проходя через отверстие, не встречает сопротивления.

    Когда сила прикладывается к сжатой жидкости, жидкость ищет путь наименьшего сопротивления.

    Неисправности оборудования, использующие давление масла.

    Вышеописанные характеристики гидравлических жидкостей являются полезными для гидравлического оборудования, но также являются источником многих неисправностей.

    Например, если произошла течь в системе, гидравлическая жидкость будет вытекать, так как ищет путь наименьшего сопротивления. Типичными примерами является течь ослабленных соединений и уплотнений. 

    Естественное давление

    Мы разговаривали про давление и поток, но часто давление существует без потока. Сила тяжести является хорошим примером. Если мы имеем три взаимосвязанных резервуара разного уровня, как показано на рисунке, сила тяжести сохраняет жидкости во всех резервуарах на одном уровне. Это другой принцип, который мы можем использовать в гидравлической системе.

    Значение силы тяжести

    Под действием силы тяжести масло попадает из бака к насосу. Масло не всасывается насосом, как думают многие люди. Насос служит для подачи масла. Что обычно понимают под всасыванием насоса, обозначает подачу масла к насосу под действием силы тяжести.

    Масло к насосу поступает под действием силы тяжести.

    Масса жидкости

    Масса жидкости также создаёт давление. Дайвер, который ныряет в море, скажет, что он не может нырять слишком глубоко. Если дайвер опустится слишком глубоко, давление раздавит его. Это давление создаётся массой воды. Таким образом, мы имеем вид давления, которое появляется самостоятельно от веса воды.

    Давление возрастает пропорционально глубине и мы можем точно измерить давление на глубине. На рисунке изображена квадратная колонна с водой высотой 10 метров. Известно, что один кубический метр воды весит 1000 кг. При увеличении высоты колонны до 10 метров, вес колонны увеличится до 10000 кг. На дне образуется один квадратный метр. Таким образом вес распределяется на 10000 квадратных сантиметров. Если мы разделим 10000 кг на 10000 квадратных сантиметров, то получится, что давление на этой глубине составляет 1 кг на 1 квадратный сантиметр.

    Что вызывает давление?

    Когда давление смешивается с потоком, мы имеем гидравлическую силу. Откуда поступает давление в гидравлическую систему. Часть — это результат силы тяжести, но откуда берётся остальное давление.

    Нагрузка создает давление. Большая часть давления появляется от воздействия нагрузки. На рисунке ниже, насос подаёт масло непрерывно. Масло из насоса находит путь наименьшего сопротивления и направляется через шланг к рабочему цилиндру. Вес нагрузки создаёт давление, величина которого зависит от веса.

    Давление в параллельном соединении

    Имеется три различных груза, соединённых параллельно в одной гидравлической системе, как показано на рисунке ниже. Масло, как обычно, ищет путь наименьшего сопротивления. Это значит, что самый лёгкий груз поднимется первым, потому что цилиндру В понадобится наименьшее давление. Когда самый лёгкий груз поднимется, давление возрастёт, чтобы поднять следующий по весу груз из оставшихся. Когда цилиндр А достигнет окончания хода, давление возрастёт, чтобы поднять самый тяжёлый груз. Цилиндр С поднимется последним.

    Гидравлическая сила рабочего цилиндра

    (1) Закон инертности говорит о том, что свойство тела сохранять своё состояние покоя или прямолинейного равномерного движения, пока какая-либо внешняя сила не выведет его из этого состояния. Это одна причина, почему поршень рабочего цилиндра не двигается.

    (2) Другая причина, почему поршень не двигается это нахождение на нём груза.

    (3) Когда насос начинает давить на цилиндр, рабочий поршень и груз оказывают сопротивление потоку масла. Таким образом, давление возрастает. Когда это давление преодолевает сопротивление поршня, поршень начинает движение.

    (4) Когда поршень двигается вверх, он поднимает груз. Давление и поток используются вместе для выполнения работы. Это гидравлическая сила в действии.

    Поток

    Ранее мы говорили, что поток совершает работу и двигает предметы. Имеется другой ключевой момент — Каким образом скорость потока относится к работе гидравлической системы?

    Ответом является то, что скорость потока постоянная.

    Возрастающая скорость потока создаёт высокую скорость

    Многие люди думают, что возрастающее давление повышает скорость, но это не правда. Вы не можете заставить двигаться поршень быстрее, повысив давление. Если вы хотите заставить двигаться поршень быстрее, вы должны повысить скорость потока.

    При закрытие предохранительного клапана, скорость не возрастает

    Здесь приведена одна распространённая ошибка при поиске неисправности в гидравлической системе. Когда скорость цилиндра падает, некоторые механики сразу направляются к предохранительному клапану, потому что они думают, что повышение давления увеличит рабочую скорость. Они стараются уменьшить настройки предохранительного клапана, что предполагается повысит максимальное давление в системе. Такие изменения не приводят к увеличению скорости действия. Предохранительный клапан служит для защиты гидравлической системы от чрезмерного давления. Параметры давления никогда не должны быть выше величины установленного давления. Вместо повышения установок давления, механики должны искать другие причины неисправности системы.

    Парциальное давление и его формула

    Хотя формула гидростатического давления применима для газов, но давления для них удобнее вычислять по другой формуле, формуле парциального давления.

    Дело в том, что в природе редко встречаются абсолютно чистые вещества, причем это касается как жидкостей, так и газов. Обычно на практике в окружающем мире преобладают различные смеси, и логично, что каждый из компонентов такой смеси может оказывать разное давление, такое разное давление и называют парциальным. Определить парциальное давление просто – оно равно суме давлений каждого компонента рассматриваемой смеси. Отсюда формула парциального давления будет иметь следующий вид:

    P = P1+P2+P3

    Где P1, P2 и P3 – давления каждого из компонентов газовой смеси, так званный «идеальный газ».

    К примеру, чтобы определить давления воздуха обычной формулы гидростатического давления проделанной только с

    кислородомнедостаточно, так как воздух в реальности представляет собой смесь разных газов, где помимо основного компонента кислорода, которым мы все дышим, есть и другие: азот, аргон и т. д.

    Такие расчеты нужно проделывать при помощи формулы парциального давления.

    Приборы для измерения давления

    Разумеется, человечество изобрело многие приборы, позволяющие быстро и удобно измерять уровень давления. Для измерения давления окружающей среды, оно же атмосферное давление используют такой прибор как манометр или барометр.

    барометр

    Так выглядит классический барометр для измерения атмосферного давления.

    Чтобы узнать артериальное давление у человека, часто служащее причиной недомоганий используется прибор известный большинству под названием неинвазивный тонометр. Таких приборов существует множество разновидностей.

    Также биологи в своих исследованиях занимаются расчетами осмотического давления – это давление внутри и снаружи клетки. А метеорологи, в частности по перепадам давления в окружающей среде предсказывают нам погоду.

    Абсолютное

    Так называется полное давление, под которым находится вещество или объект, без учета влияния других газообразных составляющих атмосферы.

    Измеряется оно в паскалях и являет собою сумму избыточного и атмосферного давлений. Также он является разностью барометрического и вакуумметрического видов.

    Вычисляется оно по формуле Р = Р2 + Р3 или Р = Р2 — Р4.

    За начало отсчета для абсолютного давления в условиях планеты Земля, берется давление внутри емкости, из которой удален воздух (то есть классический вакуум).

    Только такой вид давления используется в большинстве термодинамических формул.

    «Как рассчитать перепад давления?», в этой статье мы постараемся найти ответ на этот вопрос. Также мы обсудим различные устройства для измерения перепада давления.

    Разность давлений, также известная как градиент давления или дифференциальное давление, представляет собой разницу между двумя измеренными значениями давления или сравнение двух произвольных давлений.

    Перепад давления рассчитывается в различных отраслях промышленности на ежедневной основе для потока и фильтрации различных жидкостей по трубопроводам.

    Разность давлений или дифференциальное давление ΔP представляет собой разницу между любыми двумя приложенными давлениями, как правило, все показания давления можно считать дифференциальными, как, в случае абсолютного давления, по отношению к вакуумметрическому давлению и манометрическому давлению по отношению к атмосферному давлению.

    Водяное давление; Кредит изображения:unsplash
    Утечка трубы высокого давления; Кредит изображения:unsplash

    Что такое давление?

    Проще говоря, давление — это сила, действующая на заданную площадь.

    Давление определяется как сила на единицу площади.

    Давление = сила/площадь

    Где Сила (F) находится в Ньютонах

    И площадь (A) в метрах2

    Следовательно, давление, p=F/A, ньютон/метр.2 или Паскаль.

     Давление увеличивается по мере того, как мы спускаемся с поверхности жидкости, давление всегда больше для более плотной среды. Манометр и барометр — это измерительные приборы, используемые для измерения давления.

    как рассчитать разницу давлений

    Давление, возникающее при столкновении частиц внутри закрытого контейнера; Изображение Фото: википедия
    Давление = сила/площадь; Кредит изображения: википедия

    Что такое разница давлений?

    Перепад давления рассчитывается через равные промежутки времени или непрерывно в различных отраслях промышленности, таких как химические заводы, нефтеперерабатывающие заводы, нефтехимические заводы и т. д.

    Разность давлений – это разница между давлениями, когда они измерены в разных точках. Расчеты перепада давления регулярно выполняются в различных отраслях промышленности для измерения расхода и уровня жидкостей, мониторинга фильтров и обнаружения засоров. Разница давлений измеряется в фунтах на квадратный дюйм (PSI).

    Эта простая концепция перепада давления играет важную роль в перерабатывающей промышленности, поскольку она отвечает за поток жидкости из одной точки в другую. Повышение давления в кабинах самолетов является одним из наиболее популярных применений перепада давления.

    Устройство давления для промышленной системы; Кредит изображения:unsplash

    Перепад давления постоянно контролируется в промышленности, чтобы знать, есть ли какие-либо засоры или загрязнения в трубопроводе.

    Если часть трубопровода забивается, происходит изменение давления, что приводит к изменению перепада давления, оператор может легко определить точную точку засорения и очистить трубопровод, чтобы обеспечить плавный поток жидкости.

    Для максимального значения перепада давления нам, возможно, придется заменить фильтр, иначе это может привести к повреждению или коррозии.

    Для определения дыхательного потока в медицинских устройствах, блокировки потока воздуха в различных устройствах, обнаружения засорения фильтров в вытяжных вентиляторах, кондиционерах, вытяжных каналах — это примеры приложений, где мониторинг перепада давления играет важную роль.

    Разница гидростатического давления

    Разность гидростатического давления — это конкретное измерение давления жидкости над вертикальной исходной точкой в ​​разных точках или на разной высоте от исходной линии.

    Давление жидкости в состоянии покоя на определенной глубине из-за силы гравитации известно как гидростатическое давление, и гидростатическое давление увеличивается по мере увеличения глубины, измеренной от поверхности, из-за того, что увеличивающийся вес жидкости оказывает большее нисходящее усилие на нижние слои жидкости.

    Гидростатическое давление на разных глубинах; Кредит изображения: википедия

    Формула, используемая для расчета гидростатического давления, показана ниже:

    р=ρgh

    Где,

    P — давление, оказываемое жидкостью в Нм.-2

    ρ — плотность жидкости в кгм-3

    g — ускорение свободного падения, рассматриваемое как 9.81 мс.-2

    h — высота столба жидкости в м.

    Пловец испытывает большее давление по мере того, как он уходит глубже под воду, из-за все большего и большего веса на поверхности, а также воздуха над ним.

    Как рассчитать разницу давлений?

    .Дифференциальные манометры и дифференциальные U-образные манометры используются для расчета разницы давлений между двумя точками одной и той же жидкости или разных жидкостей.

    Дифференциальные манометры состоят из двух впускных отверстий, каждое из которых соединено с одной из труб или источников, давление в которых должно контролироваться.

    Манометр с U-образной трубкой — это устройство, используемое для измерения разницы давлений между двумя точками, которые могут находиться в одной и той же трубе или в двух разных трубах. Как следует из названия, это похоже на букву U, нижняя часть стеклянной трубки заполнена тяжелой жидкостью, такой как ртуть, вода и воздух также используются в соответствии с требованиями.

    Дифференциальный ртутный манометр; Кредит изображения:википедия
    Манометр в действии; Кредит изображения:википедия

    Манометр дифференциального давления выполняет математическую операцию вычитания с помощью механических средств. Если существует разница давлений как в трубах, так и в источниках, давление, оказываемое на один измерительный элемент, больше, чем давление, оказываемое на другой элемент камеры, тогда манометр дифференциального давления показывает отклонение.

    Как рассчитать перепад давления в манометре?

    Разность давлений между двумя трубами или источниками рассчитывается с помощью дифференциального U-образного манометра.

    Дифференциальный манометр — это устройство, используемое для сравнения давлений в двух разных трубах или емкостях по вертикальному расстоянию от поверхности жидкости в двух патрубках, когда каждый патрубок подключен к двум разным источникам.

    U-образный манометр; Кредит изображения: википедия

    Чтобы определить разницу давлений, умножьте разницу высот на плотность жидкости и ускорение свободного падения. Конечные единицы должны быть в паскалях.

    Как рассчитать расход по перепаду давления?

    Уравнение Бернулли дает связь между разностью давлений и Поток жидкости, используя это соотношение, мы можем легко рассчитать количество жидкости, протекающей в течение интервала времени.

    Принцип Бернулли применим для несжимаемых жидкостей (с пренебрежимо малым вязкость) утверждает, что увеличение скорости жидкости, движущейся по линии тока, происходит при уменьшении статического давления жидкости. Проще говоря, статическое давление + динамическое давление = постоянное общее давление.

    Поток жидкости через вентуриметр; Кредит изображения:википедия
    Поток жидкости через вентуриметр; Кредит изображения:википедия

    Принцип Бернулли можно выразить в виде математической формулы, как показано ниже:

    Где v=скорость жидкости

    g = ускорение свободного падения, рассматриваемое как 9.81 мс.-2

    y=высота над базовой линией

    p=давление жидкости

    ρ = плотность жидкости

    Теперь рассмотрим любые две точки в жидкости,

    Теперь g одинаково в обеих точках,

    Так как жидкость считается несжимаемой,

    Из уравнения непрерывности, v1A1 = V2A2

    Из уравнения (2)

    Из уравнения (3) мы можем рассчитать скорость потока (Q = площадь x средняя скорость) через воздуховод, используя разницу давлений p1 — п2

    Измерение количества потоки жидкости в пределах временного интервала очень важно в перерабатывающей промышленности для бесперебойного и безопасного выполнения различных операций.

    Уравнение Хагена-Пуазейля дает отношения между падение давления и расход расход жидкости через длинную цилиндрическую трубу. Уравнение применяется для ламинарный поток несжимаемой жидкости, протекающей по трубе постоянного сечения.

    Где Δp — разница давлений между двумя концами трубы.

    L – длина трубы,

    μ — динамическая вязкость,

    объемный скорость потока,

    R — радиус трубы,

    А — поперечное сечение трубы.

    Как рассчитать перепад давления в расходомере Вентури?

    Вентуриметр — тип расходомера, в котором объемный расход жидкости определяется на основе теоремы Бернулли.

    Когда жидкость проходит через вентуриметр, она ускоряется в сужающейся части и затем замедляется в расширяющейся части. Давление жидкости в вентуриметре разное, и разность давлений получается из показаний манометра. Используя эту расчетную разницу давлений и применяя уравнение Бернулли и уравнение непрерывности, рассчитывается объемный расход.

    как рассчитать разницу давлений

    Вентуриметр; Кредит изображения: википедия
    Вентуриметр; Кредит изображения: википедия

    Что нужно знать о скорости потока(кликните сюда)

    Понравилась статья? Поделить с друзьями:
  • Как найти код на телевизоре самсунг
  • Как найти матрицу перехода онлайн
  • Как правильно составить учетную политику на 2019 год
  • Как найти женщину которая будет содержать меня
  • Как составить ролик для радио